Download NR509 FINAL EXAM TEST BANK WITH ACTUAL
CORRECT QUESTIONS AND VERIFIED DETAILED
RATIONALE and more Exams Nursing in PDF only on Docsity! Page | 1 NR509 FINAL EXAM TEST BANK WITH ACTUAL CORRECT QUESTIONS AND VERIFIED DETAILED RATIONALES ANSWERS BY EXPERTS |FREQUENTLY TESTED QUESTIONS AND SOLUTIONS | 2024 NEWEST |ALREADY GRADED A+|LATEST UPDATE A 24-year-old graphic designer presents to clinic with a concern for a breast mass. A rubbery, mobile, nontender mass is palpated in the right breast as described by the patient, which is consistent with a firbroadenoma. In describing the location of the mass, the examiner notes that it is 3 cm proximal to and 3 cm to the left of the nipple. Which of the following would be the most appropriate way to report this finding? a. "Rubbery, mobile, nontender mass located in right breast, in the 10:30 position from the nipple" b. "Rubbery, mobile, nontender mass located in right breast, in the lower outer quadrant" c. "Rubbery, mobile, nontender mass located in right breast, in the upper inner quadrant" d. "Rubbery, mobile, nontender mass located in the left breast, upper outer quadrant" e. "Rubbery, mobile, nontender mass located in right breast, in the 1:30 position from the nipple" a. "Rubbery, mobile, nontender mass located in right breast, in the 10:30 position from the nipple" A 54-year-old female dietician presents for a routine annual examination. On review of systems, she reports that she has had many breast findings over several years, including one biopsy with normal pathology. She feels that her breasts have become far less lumpy since she underwent menopause 3 years ago. Which of the following is true regarding changes in the breasts with menopause? a. Transformation of breasts to primarily fatty tissue with menopause decreases the sensitivity and specificity of mammograms. b. Estrogen in hormone replacement therapy (HRT) has no effect on breast density after menopause. c. Glandular tissue of the breast atrophies with menopause, primarily due to decrease in the number of lobules. d. Breast density has no genetic component and is entirely due to estrogen dose from endogenous and exogenous sources over the lifetime. e. Mammography performs most poorly in the menopausal and postmenopa c. Glandular tissue of the breast atrophies with menopause, primarily due to decrease in the number of lobules. An overweight 26-year-old public servant presents to the Emergency Department with 12 hours of intense abdominal pain, light-headedness, and a fainting episode that finally prompted her to seek medical attention. She has a strong family history of gallstones and is concerned about this possibility. She has not had any vomiting or diarrhea. She had a normal bowel movement this morning. Her Page | 2 βhuman chorionic gonadotropin (β-hCG) is positive at triage. She reports that her last period was 10 weeks ago. Her vital signs at triage are pulse, 118; blood pressure, 86/68; respiratory rate, 20/min; oxygen saturation, 99%; and temperature, 37.3ºC orally. The clinician performs an abdominal exam prior to her pelvic exam and, on palpation of her abdomen, finds involuntary rigidity and rebound tenderness. What is the most likely diagnosis? a. Ruptured tubal (or ectopic) pregnancy b. Acute cholecystitis c. Ruptured appendix d. P a. Ruptured tubal (or ectopic) pregnancy A 63-year-old janitor with a history of adenomatous colonic polyps presents for a well visit. Basic labs are performed to screen for diabetes mellitus and dyslipidemia. Electrolytes and liver enzymes were also measured. His labs are all normal expect for moderate elevations of aspartate aminotransferase, alanine aminotransferase, γ-glutamyl transferase, and alkaline phosphatase as well as a mildly elevated total bilirubin. He presents for a follow-up appointment and the clinician performs an abdominal exam to assess his liver. Which of the following findings would be most consistent with hepatomegaly? a. Liver span of 11 cm at the midclavicular line b. Liver span of 8 cm at the midsternal line c. Dullness to percussion over a span of 11 cm at the midclavicular line d. Dullness to percussion over a span of 8 cm at the midsternal line e. Liver palpable 3 cm below the right costal margin, mid clavicular line, on expir e. Liver palpable 3 cm below the right costal margin, mid clavicular line, on expiration A 63-year-old underweight administrative clerk with a 50-pack-year smoking history presents with a several month history of recurrent epigastric abdominal discomfort. She feels fairly well otherwise and denies any nausea, vomiting, diarrhea, or constipation. She reports that a first cousin died from a ruptured aneurysm at age 68 years. Her vital signs are pulse, 86; blood pressure, 148/92; respiratory rate, 16; oxygen saturation, 95%; and temperature, 36.2ºC. Her body mass index is 17.6. On exam, her abdominal aorta is prominent, which is concerning for an abdominal aortic aneurysm (AAA). Which of the following is her most significant risk factor for an AAA? a. Female gender b. History of smoking c. Underweight d. Family history of ruptured aneurysm e. Hypertension b. History of smoking A 76-year-old retired man with a history of prostate cancer and hypertension has been screened annually for colon cancer using high sensitivity fecal occult blood testing (FOBT). He presents for followup of his hypertension, during which the clinician scans his chart to ensure he is up to date with Page | 5 d. Seminal vesicle e. Vas deferens a. Corpora cavernosa A 29-year-old graduate student states that he is able to achieve an erection and ejaculate during sexual intercourse; however, he does not experience any pleasurable sensation of orgasm. He is otherwise healthy and is not on any medications. What is the most likely cause of his problem? a. Androgen insufficiency b. Endocrine dysfunction c. Peyronie disease d. Psychogenic e. Sexually transmitted infection (STI) d. Psychogenic Multiple processes must take place in order for a male to sustain an erection. Various cues stimulate sympathetic outflow from higher brain centers to the T11-L2 levels of the spinal cord and parasympathetic outflow from S2 to S4 reflex arcs. Local vasodilatation within the penis erectile tissue results from increased levels of which of the following? a. Follicle-stimulating hormone (FSH) b. Gonadotropin-releasing hormone (GRH) c. Luteinizing hormone (LH) d. Nitric oxide (NO) and cyclic guanosine monophosphate (cGMP) e. Testosterone d. Nitric oxide (NO) and cyclic guanosine monophosphate (cGMP) The human papillomavirus (HPV) can cause genital warts in males and females as well as cervical cancer in females. Vaccination against HPV is available and should be offered to males between what ages? a. 6-9 months b. 1-3 years c. 5-7 years d. 9-21 years e. 30-50 years d. 9-21 years A 32-year-old male complains of a painless, cystic mass just above his left testicle. During the physical examination, a strong flashlight is placed behind the scrotum through the area in question and transillumination is noted. What is the most likely diagnosis? a. Direct hernia b. Indirect hernia c. Spermatocele Page | 6 d. Testicular tumor e. Varicocele c. Spermatocele A 25-year-old graduate student presents to the clinic complaining of scrotal pain, which has been increasing over the past 2 days. He is sexually active and has had unprotected intercourse with multiple partners in the past couple of weeks. On examination, some mild to moderate swelling of the scrotum on the right and tenderness with palpation of the right testicle are notes. What is the most likely diagnosis? a. Acute epididymitis b. Hydrocele c. Primary syphilis d. Spermatocele e. Testicular cancer a. Acute epididymitis A 32-year-old elementary teacher requests a workup for infertility. He and his wife have been trying to conceive for the last 2 years. He reports that his wife has been evaluated and does not appear to have any infertility issues. The overall examination does not reveal any significant abnormalities. He is of average height and weight and has normal secondary sex characteristics of the genitalia. Of the following, which would be most likely be abnormal and causing male infertility? a. 5α-Dihydrotestosterone b. 5α-Reductase c. Follicle-stimulating hormone (FSH) d. Luteinizing hormone (LH) e. Thyroid-stimulating hormone (TSH) c. Follicle-stimulating hormone (FSH) While performing a physical examination on male patients, it is possible to palpate multiple structures in relation to the inguinal canal and related hernias. Which of the following is not palpable during an external examination of the abdominal wall or inguinal region? a. External inguinal ring b. Internal inguinal ring c. Pubic tubercle d. Anterior superior iliac spine e. Direct inguinal hernia b. Internal inguinal ring A 20-year-old college student presents for his annual physical examination. He recently became sexually active and is inquiring about the best means of preventing sexually transmitted infections (STIs). Of the following, which would be the most effective means of prevention? Page | 7 a. Early withdrawal b. Male condoms c. Spermicides d. Diaphragms e. Cervical caps b. Male condoms A 21-year-old college student presents to the student health clinic for a full physical examination. He is generally healthy; however, he reports that he has had sexual intercourse with multiple partners in the past couple of months. He noticed a small lesion on the shaft of his penis a few days ago. While performing the examination, he unwillingly achieves an erection. How should the clinician proceed at this point? a. Stop the examination immediately. b. Have him return to see another provider. c. Explain this is a normal response and finish the examination. d. Tell him the examination cannot proceed until the erection subsides. e. Assume that he is malingering. c. Explain this is a normal response and finish the examination. A 45-year-old driver's education instructor presents to the clinic for heavy periods and pelvic pain during her menses. She reached menarche at age 13 years and has had regular periods except during her pregnancies. She is a G4P3013 and does not use birth control as her husband has had a vasectomy. She states this has been going on for about a year but seems to be getting worse. Her last period was 1 week ago. On bimanual exam, a large midline mass halfway to the umbilicus is palpated. Each adnexal area is nonpalpable. Her rectal exam is normal. Her body mass index (BMI) is 27. What is the best explanation for her physical finding? a. Large colonic stool b. Ovarian mass c. Fibroids d. 4-Month pregnancy e. Bartholin gland enlargement c. Fibroids A 32-year-old G0 woman comes for evaluation on why she and her husband have been unable to get pregnant. Her husband has been married before and has two other children, ages 7 and 4 years. The patient relates she began her periods at age 12 and has been fairly regular ever since. She began oral contraceptive pills from when she got married until last year, when she began to try for a pregnancy. Before this she had regular cycles for 10 years. She has had a history of five prior partners. She relates she was once treated for a severe genital infection when she was in college. Based on this patient's history, what is the best explanation for her infertility? Page | 10 c. Withdraw the speculum slightly and reposition it on a different slope A 63-year-old office worker comes to the clinic for her women's health exam. Her last Pap smear was 5 years ago and was normal. She is married and has been with the same sexual partner for the last 35 years. After performing the majority of the exam, the clinician decides to do a speculum exam to collect cytology for Pap smear. What is the correct position to have the patient in for her speculum exam? a. Sitting b. Supine c. Prone d. Trendelenburg e. Lithotomy e. Lithotomy A 68-year-old retired patient presents to the clinic complaining about feeling like something is falling out of her vagina. She is a G6P6007 and had all her children vaginally, even the twins. She went through menopause at age 55 years, and, for the last few months, she has felt this falling sensation. On exam, an anterior bulge in the vaginal wall is apparent when she bears down. Weakness in which muscle would best account for the anterior bulge in the vaginal wall? a. Levatori ani b. Anal sphincter c. Pubis symphysis d. Ischiocavernosus muscle e. Bulbocavernosus muscle a. Levatori ani A 49-year-old male nurse experiences fecal incontinence after a motor vehicle accident that left him paralyzed below the waist. He asks his rehabilitation physician about the control of this function in a person without his injuries. Which of the following is true regarding the muscle control of the anal sphincter? a. The internal anal sphincter is under voluntary control, whereas the external anal sphincter is under involuntary control. b. The internal anal sphincter is under involuntary control, whereas the external anal sphincter is under voluntary control. c. Both internal and external anal sphincter are under voluntary control. d. Both internal and external anal sphincter are under involuntary control. e. Control of the anal sphincters is variable between individuals. b. The internal anal sphincter is under involuntary control, whereas the external anal sphincter is under voluntary control. A 62-year-old male who is undergoing evaluation for possible prostate cancer strongly declines a rectal examination, stating that, "Some trainee once did that and it hurt badly." Which of the following is true Page | 11 about the innervation of the anus and rectum that may explain this patient's experience of discomfort? a. The rectum contains primarily somatic nerves, whereas the anal canal contains primarily visceral nerves, making the rectum the most likely source of this patient's discomfort. b. The rectum contains primarily somatic nerves, whereas the anal canal contains primarily visceral nerves, making the anus the most likely source of this patient's discomfort. c. Proximal to the dentate line, the lower gastrointestinal tract is innervated primarily by somatosensory nerves, making the proximal reach of the examination the most likely site of this patient's pain d. The anal canal has a rich somatosensory innervation, d. The anal canal has a rich somatosensory innervation, making poorly directed examinations painful in this area. A 54-year-old male with a strong family history of breast and prostate cancer presents to his primary care provider to discuss prostate screening. His father died at age 73 years from prostate cancer that was not detected on routine digital rectal examinations (DREs), and he would like to minimize his chance of a similar occurrence. Which of the following is true regarding the anatomy and screening of the prostate by DRE? a. All three lobes of the prostate are palpable on DRE. b. The seminal vesicles are palpable distal to the prostate on DRE. c. The median lobe of the prostate is located anterior to the urethra and is not palpable on DRE. d. The median sulcus divides the lateral lobes from the median lobe and is palpable on DRE. e. A prostate of 5 cm diameter without palpable nodes or masses represents a normal prostate examination. c. The median lobe of the prostate is located anterior to the urethra and is not palpable on DRE. A third-year medical student rotating on the internal medicine service performs a digital rectal examination (DRE) on a 56-year-old female patient. The patient has been admitted for suspicion of a myocardial infarction, and confirmation that there is no blood in the stool is required before anticoagulation can be started. The student reports that the fecal occult blood test was negative but notes that he palpated a structure through the anterior rectum that he could not identify. The attending physician confirms normal anatomy and reviews with the student that the most likely identity of the structure palpable is which of the following a. Sacrum b. Pectinate line c. Uterine fundus d. Prostat e. Cervix e. Cervix A 45-year-old female executive reports to her primary care provider that she has recently experienced a change in the patterns of her bowel movements. She expresses a great concern as her family history includes a maternal aunt who died of colon cancer at age 49 years; her mother has had colonoscopies Page | 12 every 3 years with numerous adenomatous polyps removed. Which of the following historical elements would be the most concerning for colon cancer in this patient? a. Long-term history of hemorrhoids b. Recent history of black, tarry stools c. Remote history of anal pruritus d. New-onset anal fissure e. Recent onset of small-caliber stools e. Recent onset of small-caliber stools A 49-year-old customer service representative presents to his gastroenterologist for follow-up of his long-standing inflammatory bowel disease (IBD). He was diagnosed with ulcerative colitis (UC) at age 37 years and has had irregular care for this condition since then. His sole colonoscopy was done at the time of diagnosis 12 years ago. His only relevant family history is of prostate cancer in his father; his mother and sisters are healthy. Which of the following is true about recommended screening for colon cancer in this patient? a. The patient should begin screening for colon cancer 10 years prior to the age of onset of his father's prostate cancer. b. The patient should undergo colonoscopy for his bowel condition, which confers risk of colon cancer. c. The patient is due for routine age-based colon cancer screening by colonoscopy regardless of his risk factors. d. The patient has a reassuring family history and b. The patient should undergo colonoscopy for his bowel condition, which confers risk of colon cancer. A 49-year-old male with well-controlled HIV undergoes a proctoscopic examination as routine screening for anal cancer. The patient is asymptomatic and specifically denies complaints of frequent urination (frequency), large volume of urination (polyuria), or repeated urination at night (nocturia). Under direct visualization, the clinician observes a clear, circumferential demarcation of proximal versus distal tissue. This demarcation was not palpable on digital rectal examination (DRE) prior to proctoscopy. What is the most likely origin of this finding? a. Pathological constriction of the anal canal b. Normal anatomy of the mucosal surface c. Carcinoma d. Valve of Houston e. External anal sphincter b. Normal anatomy of the mucosal surface A 34-year-old female reports anal pain with defecation. She notes incidentally to this complaint that she has developed episodic abdominal discomfort and sores in her mouth. Anoscopic examination reveals anal fissures that appear to be her source of pain. Which of the following underlying conditions is the clinician most likely to find? Page | 15 c. Compare the strength of his trapezia muscles d. Assess his ability to extend his back e. Check for listing of his trunk c. Compare the strength of his trapezia muscles During a musculoskeletal examination, the clinician instructs the patient to look over one shoulder, and then the other shoulder. This action assesses the movement of which muscle(s)? a. Scalenes b. Sternocleidomastoid (SCM) c. Splenius capitis d. Prevertebral muscles e. Splenius cervicis b. Sternocleidomastoid (SCM) During a musculoskeletal examination of the spine, what is the action(s) of the erector spinae muscle group? a. Rotation of the spine b. Extension of the spine c. Flexion of the spine d. Lateral bending of the spin e. Rotation and lateral bending of the spine b. Extension of the spine The clinician is seeing a middle-aged patient who has a diagnosis of lumbar spinal stenosis. The patient's history is consistent with this diagnosis as he has pain in the back with walking that improves with rest. Which physical sign(s) are most consistent with his diagnosis? a. Hyperreflexia of the lower limb b. Pelvic tilt or drop c. Thoracic kyphosis d. Positive straight-leg raise e. Flexed forward posture with lower extremity weakness e. Flexed forward posture with lower extremity weakness The clinician is seeing a 58-year-old patient with a diagnosis of arthritis. The patient complains of pain in his knees, hips, hands, wrists, neck, and low back. Based on which joints are involved, the patient most likely has which joint problem? a. Osteoarthritis (OA) b. Rheumatoid arthritis (RA) c. Psoriatic arthritis Page | 16 d. Gout e. Polymyalgia rheumatica a. Osteoarthritis (OA) A 62-year-old patient with rheumatoid arthritis (RA) complains of increased joint stiffness. What characteristic(s) are consistent with her diagnosis of RA? a. Stiffness follows joint activity. b. Joint distribution is asymmetrical. c. Swelling of the synovial tissue is seen in joints and tendon sheaths. d. It most frequently involves the first metatarsophalangeal joint. e. Tophi are found in the subcutaneous tissue. c. Swelling of the synovial tissue is seen in joints and tendon sheaths. A 55-year-old woman with a headache explains to the clinician that she has had headaches before, but this one is unusual because of some new symptoms. Which of the following symptoms would prompt an immediate investigation? a. The headache is similar in nature to prior ones she has had for decades but more severe. b. The patient had a car accident and minor head trauma about 3 months ago. c. The patient also has developed fever and night sweats and thinks she lost some weight. d. The headache comes and goes. e. The patient lost her glasses. c. The patient also has developed fever and night sweats and thinks she lost some weight. In the case of a middle-aged female with a pounding headache, what is an effective question to ask the patient? a. Does the patient have any aura prior to the headaches? b. How old is the patient? c. Is she feeling stressed? d. Does she think she is losing her memory? e. Has she ever seen anyone with a stroke? a. Does the patient have any aura prior to the headaches? A 35-year-old female patient has had migraines for much of her adult life. Ather regular checkup, she is healthy, takes no medications except oral contraceptive pills (OCPs), exercises, and has a steady job. Her only complaint is that her migraines seem to have become worse, and, for the past few weeks, she has been waking up at night with headache and also nausea. Which of the following is the best course of action? a. Reassure her that this is a common pattern with migraines. b. Order studies to evaluate potential transient ischemic attacks (TIAs) because she is on OCPs. Page | 17 c. Take a further history and perform a very careful neurological examination. d. Treat her for sinusitis. e. Prescribe a strong medication for her migraines. c. Take a further history and perform a very careful neurological examination. d. Treat her for sinusitis. An 82-year-old grandmother presents to the Emergency Department in the care of her extended family with new-onset speech impairment. According to family members, the patient awoke with this symptom as well as difficulty in understanding questions or following commands. Her past medical history is remarkable for atrial fibrillation but no other notable conditions. On examination, her speech is verbose but poorly comprehensible and lacks meaning. She is unable to follow simple commands. Which of the following best describes her speech disorder? a. Dysphonia with expressive deficit b. Dysarthria c. Wernicke aphasia d. Global aphasia e. Broca aphasia c. Wernicke aphasia A 74-year-old bus driver is delivered to the hospital via emergency transport after an astute passenger noted that the patient exhibited drooping facial features and slurred speech. The patient was diagnosed rapidly with ischemic (nonhemorrhagic) stroke, and urgent intervention lead to a near complete recovery from his symptoms. The astute passenger was thanked and congratulated for recognizing the signs of acute stroke; this individual credited this recognition to a public safety awareness campaign that outlined the critical public health need to recognize strokes early. Which of the following statements is true for risks and rapid recognition of suspected strokes? a. Atrial fibrillation is not a risk factor for ischemic stroke in individuals age ≥75 years. b. Hypertension is the leading risk factor for both ischemic and hemorrhagic stroke. c. Obesity with normal glucose tolerance is not a risk factor for stroke. b. Hypertension is the leading risk factor for both ischemic and hemorrhagic stroke. A 70-year-old male presents to the Emergency Department accompanied by his wife, who is concerned that he has experienced a stroke. She states that he awoke with drooping of the right side of his mouth. He has a history of hypertension and diet-controlled diabetes, but no history of prior transient ischemic attacks (TIAs), strokes, or neurologic deficits. Physical examination reveals a wellnourished, right-handed male, who has an obvious flattening of the right nasolabial fold at rest. He is unable to close his right eye, wrinkle his forehead, or raise his eyebrows. The remainder of the neurologic examination is symmetric with intact strength and normal deep tendon reflexes. Based on this history and physical examination, which of the following statements is true? a. The patient most likely has a central upper motor neuron lesion involving cranial nerve (CN) VII (the facial nerve). b. The patient most likely has a Page | 20 b. Brainstem c. Lumbar spinal cord d. Peripheral nerve e. Distal muscle d. Peripheral nerve A mother brings her 15-month-old toddler to the clinic for his preventive health care visit. The clinician takes the history and observes the child's interactions and behaviors and is then ready to begin the rest of the examination. Which of the following best describes the general approach to the pediatric examination of the young child? a. Always give immunizations prior to beginning the examination. b. Examine the child in the same order as for an adult patient. c. Children age <2 years do not need to be examined. d. Begin with least invasive parts of the examination first. e. Never examine a young child in the mother's lap. d. Begin with least invasive parts of the examination first. In caring for children, physicians and other clinicians need to understand child development. Of the following, which is a principle of normal child development? a. Child development proceeds along a predictable pathway in a healthy child. b. There is minimal variation in when children achieve milestones. c. All delays in development can be explained by one or two risk factors. d. Regression in developmental skills is not a cause for concern. e. A child's developmental level can be ignored in conducting an examination. a. Child development proceeds along a predictable pathway in a healthy child. The parents of a 21-month-old child explain that their son used to speak nearly 50 words and was using 2-word phrases. In the last month or so, the child has not been using as many words and tends to echo what is being said to him rather than use language spontaneously. They want to know if this is normal. After taking a thorough developmental history, the clinician finds that the child makes poor eye contact and does not play with toys in a purposeful manner. The physical examination is normal except for the child's limited social interactions. There is a family history of autism in two first cousins. Which of the following would be the best response to the parents at this time? a. Reassure the parents that all toddlers lose skills at some point in development. b. Reassure the parents that the child is fine as long as he has not lost skills in other domains. c. Send the child to the Emergency Department (ED). d. Re d. Refer the child to a developmental and behavioral pediatrician. A clinician is meeting the mother of a 5-year-old with asthma for the first time. The mother notes that the asthma has been poorly controlled and that the child has had multiple hospitalizations. The clinician Page | 21 inquires about family stressors and finds that the parents are divorced, the mother recently lost her job, and the child spent 2 months living with her grandparents who both smoke. Which of the following is the best example of the role of health promotion with this family? a. Plan less frequent pediatric visits because the family will take too much time. b. Delay immunizations because of the family stressors. c. Postpone vision and hearing screening because the child may not pass d. Reassure the parent that the family stressors are not impacting the child's asthma. e. Develop a health promotion plan that includes more frequent visits and guidance to assist family with stressors and improve the child's asthma s e. Develop a health promotion plan that includes more frequent visits and guidance to assist family with stressors and improve the child's asthma symptoms. A clinician is reading the chart of a full-term newborn whose mother had an uneventful pregnancy in the hospital for the first time on the day of birth. In reviewing the infant's chart, the clinician notes that, in the delivery room, at 5 minutes, the infant had a heart rate >100, strong respiratory effort, was crying vigorously, moving actively, and had good color except for some acrocyanosis of the hands and feet. This infant's APGAR score is closest to which of the following? a. 9 b. 7 c. 5 d. 3 e. 1 a. 9 A newborn who is floppy and limp, blue in color, with a heart rate of 60, and minimal respiratory effort has just been delivered. The infant has no grimace and only a very weak cry. What is the best immediate response to the infant in this situation? a. Discuss the infant's poor appearance with the parents who are both in the room. b. Suction the infant's mouth while waiting to calculate the 5-minute APGAR score. c. Dry the infant off and swaddle him. d. Begin neonatal resuscitation. e. Order a chest x-ray. d. Begin neonatal resuscitation. An infant is born 4 weeks preterm to a mother with a history of hypertension, severe diabetes, and alcohol abuse. The infant is noted to be small for gestational age (SGA), weighing just 1,500 g. Which of the following is the most important reason for assessing both gestational age and birth weight for any infant? a. These two factors help to anticipate certain medical and developmental problems. b. The parents should be informed of these. Page | 22 c. A SGA infant is at low risk for neonatal problems. d. A premature infant with a weight appropriate for gestational age has a very low risk for neonatal problems. e. Full-term, appropriate-for-gestational age (AGA) infants having a high risk of long-term problems. a. These two factors help to anticipate certain medical and developmental problems. A clinician arrives at the hospital several hours after the birth of a full-term infant. The infant is rooming in with her parents and appears to be doing well. There were no problems with the pregnancy, labor, or delivery. The nurse asks if the baby should be taken back to the nursery for examination. What is the best response to the nurse? a. State that it will be much more efficient to conduct the examination in the nursery. b. Note that the infant already had an examination in the delivery room and does not need another examination so soon. c. State that the infant should be examined in the presence of the parents so they can be taught about what their newborn can do. d. Note that the lighting is better in the newborn nursery. e. Refer the parents to a good book on newborns and wheel the infant back to the newborn nursery to conduct the examination. c. State that the infant should be examined in the presence of the parents so they can be taught about what their newborn can do. The nurse in the newborn nursery reports that she is concerned about Baby Boy Jones, who was born full-term by cesarean section for failure to progress. The pregnancy was complicated only by a maternal urinary tract infection in the first trimester. He had APGARs of 9 and 10 at 1 and 5 minutes, respectively, and had been doing well. However, now, on the fourth day of life, the infant has developed a tremor. Which of the following factors would cause the most concern about the tremor? a. There is a history of benign tremor in elderly family members. b. The infant also has asymmetric limb movements. c. The infant's vital signs are normal. d. The tremor is brief and only present when the infant is crying vigorously. e. The infant lies in a symmetric position with limbs flexed when relaxed. b. The infant also has asymmetric limb movements. A mother brings her 9-month-old son to the practice for the first time, concerned that he is not yet sitting by himself. After taking a careful history, the physician notes that the infant has good head control and can grasp a rattle but is unable to roll over, crawl, or pull to stand. What should the clinician explain to the mother? a. Her child is progressing normally and does not need further evaluation. b. Gross motor development proceeds from peripheral skills, such as finger feeding, to central skills, such as sitting. c. As long as the child is babbling, delays in gross motor skills are not a concern. Page | 25 vaccination should be given postpartum to protect future pregnancies from the effects of congenital rubella. b. Hepatitis B, measles/mumps/rubella (MMR), and influenza vaccines are safe during pregnancy. c. Polio and influenza vaccinations are not safe during pregnancy and should never be utilized. d. RhoGAM is a vaccine specific to pregnancy that should be given to all pregnant women e. No vaccines are safe during pregnancy, and the risks of vaccination outweigh the benefits of immunity to infectious diseases. a. If a pregnant woman does not show sufficient titers to rubella, measles/mumps/rubella (MMR) vaccination should be given postpartum to protect future pregnancies from the effects of congenital rubella. A 31-year-old marathon runner presents for prenatal care with her first pregnancy. She is in her second trimester and is experiencing some fatigue and muscle aches. Her prepregnancy body mass index (BMI) was noted at 19.2. How should she be counseled on exercise and nutrition during pregnancy? a. She should gain at least 40 pounds during the pregnancy to account for being underweight at the time of conception. b. Immersion in hot water is a safe and effective nonmedicinal way of coping with musculoskeletal complaints during pregnancy. c. She should increase her calorie intake to 300 calories per day or more from her prepregnancy baseline. d. She should avoid unpasteurized dairy products and delicatessen meats due to the risk of mycobacteria, shigellosis, and brucellosis. e. She should switch from running to weight-lifting (e.g., bench press) to maintain muscle mass while avoiding the stressors of running on the fetu c. She should increase her calorie intake to 300 calories per day or more from her prepregnancy baseline. A 29-year-old G2P1 presents to the clinic after a positive home pregnancy test. She confides at the appointment that her male partner has become increasingly abusive lately and once struck her while she was holding her older child. How should she be counseled? a. Ask that she bring the partner to all appointments so that he can be included in decisions and thus feel less threatened and less likely to harm the patient again. b. Reassure her that no matter what she reveals, all information she discloses will be kept strictly confidential. c. Ask open-ended questions, allow her to make decisions that she feels are best for herself given the circumstance, and provide immediate or long-term referrals to domestic violence resources. d. Reassure her that she is safe as very few pregnant women are hurt or murdered by their partners, who generally become less violent during the vulnerable period of pregnancy e. Demand that s c. Ask open-ended questions, allow her to make decisions that she feels are best for herself given the circumstance, and provide immediate or long-term referrals to domestic violence resources. Page | 26 A 34-year-old G3P2 at 27 weeks' gestation is referred to the clinic upon discharge from a correctional institution where she has been incarcerated for 25 days for a drug offense. She denies any further substance abuse, but her behavior is concerning for intoxication, and she smells of alcohol and cigarettes. The clinician inquires about her drug use with open-ended questions and counsels her that which of the following is true? a. Pregnant women are not routinely screened for hepatitis C, but this test should be added to the panel of prenatal blood tests for patients with a history of intravenous drug use. b. Cigarettes are a rare cause of low birth weight in the growing fetus. c. Women can safely drink one alcoholic drink per day without risk of fetal alcohol syndrome. d. If a pregnant patient does not intend to quit tobacco, she should not bother to cut down as there is no benefit to the pregnancy from decreased a. Pregnant women are not routinely screened for hepatitis C, but this test should be added to the panel of prenatal blood tests for patients with a history of intravenous drug use. A 78-year-old woman presents to clinic with her two daughters, who are concerned about her continued ability to live independently. She has thus far been highly self-reliant and is opposed to the idea of leaving of her home of 30 years. The clinician performs a complete history and physical exam (including mental status and memory testing) as well as orders laboratory tests before providing the patient and her family the finding that she has age-appropriate changes that do not reflect any particular disease process. Which of the following findings is most consistent with the normal aging process and does not impair the ability to live alone? a. Age-related cognitive decline b. Mild cognitive impairment c. Decreased level of thyroid hormone d. Persistent urinary incontinence e. Decreased adipose-to-muscle ratio a. Age-related cognitive decline Which of the following best describes the role of the health practitioner in caring for the aging American population? a. Prepare all persons age ≥65 years for the eventuality that they will become frail. b. Understand that the older population is generally homogenous with little variation in needs. c. Evaluate geriatric conditions in terms of functionality and quality of life rather than via traditional disease models. d. Employ the same disease models used to treat younger patients with chronic disease. e. Assure that all elders complete an annual physical examination. c. Evaluate geriatric conditions in terms of functionality and quality of life rather than via traditional disease models. Which of the following is true about hair in the aging adult? Page | 27 a. Age-related hair loss in males is normal only after age 50 years. b. Age-related hair loss on the scalp is abnormal in women and should be evaluated to rule out underlying pathology. c. Although hair loss occurs in both sexes, hair on the head, trunk, legs, and pubic hair is invariably spared any age-related changes. d. Women may experience the development of sparse coarse facial hair in their mid-50s. e. Age-related hair changes are the same for all individuals regardless of ethnicity or race. d. Women may experience the development of sparse coarse facial hair in their mid-50s. A 75-year-old female in generally good health presents to a new primary care provider after she recently moved to a new city. She takes no prescribed medications, but she has been told in the past that her blood pressure was borderline elevated and might require treatment at some time in the future. Which of the following findings during the physical examine is consistent with the normal aging process and not a sign of cardiovascular disease? a. A narrowed pulse pressure with increased systolic and diastolic components b. A widened pulse pressure with increased systolic pressure (up to 140) and decreased diastolic pressure c. An unchanged pulse pressure with equal increases in both systolic and diastolic pressures d. A drop in systolic pressure of 25 mm Hg when rising from a supine to standing position e. An isolated increase in systolic blood pressure to >150 mm Hg b. A widened pulse pressure with increased systolic pressure (up to 140) and decreased diastolic pressure A 66-year-old recently retired restaurant worker presents to his primary care provider with a concern about hearing loss. He relates a history of difficulty distinguishing voices in crowded settings when significant background noise exists, which hastened his retirement. Which of the following is true about this patient's experience with hearing and the aging process? a. His experience is consistent with the normal aging process. b. Any hearing impairment that causes functional decline warrants formal testing and evaluation. c. Early age-related hearing loss initially affects lower-pitched sounds. d. Decreased hearing acuity associated with aging is formally known as hypoacusis. e. Age-related declines in hearing does not begin until age ≥75 years. b. Any hearing impairment that causes functional decline warrants formal testing and evaluation. Medications carry both risks and benefits for older patients. Although the risks of polypharmacy (the use of many medications at once) are very well known, many older patients take many medications for a variety of conditions. Which of the following best describes medication prescribing and utilization in the older adult population? a. Although older patients take more medications than younger adults, their rate of hospitalization for drug-related adverse reactions is the same. b. Older patients rarely take or have adverse effects from sleep medications. Page | 30 e. Prolactinoma A 22-year-old G0P0 undergraduate student presents to clinic after finding a breast mass on breast self- examination (BSE) at home. The mass is nontender without skin changes, erythema, or overlying swelling. She has heard that most breast cancers are found by patients themselves, and she is very concerned that she may have breast cancer. Which of the following is true about BSE and self-detection of breast cancer? a. Most masses that women find at home and bring to a provider's attention turn out to be malignant. b. This patient is more likely to find a fibroadenoma than a cancer on self-examination. c. The most likely breast mass this patient is likely to find in herself is an abscess complicating underlying mastitis. d. Because of this patient's age, breast masses should not be pursued with imaging and diagnosis because the risk of cancer is so low. e. BSE is universally recommended because of very high sensitivity b. This patient is more likely to find a fibroadenoma than a cancer on self-examination. A 48-year-old female psychologist presents to clinic with concerns about her breast cancer risk after an age-matched cousin was recently diagnosed with this disease. This cousin is the third family member on her father's side in as many years to be diagnosed with breast cancer, including the patient's own father, who had surgery and subsequent treatment 3 years ago for breast cancer. The patient has little other knowledge of her family history, only that her grandparents independently arrived from Eastern Europe near the end of World War II and were among very few members of their family that survived the war. The patient has read about testing for the breast cancer genes (BRCA1 and BRCA2) and desires further information about whether this would be appropriate for her. Which of the following is true about this patient's indications for BRCA testing? a. Her familial lineage is irrelevant to her risk of BRCA genes and d. This patient carries several risk factors that together justify BRCA testing. A 68-year-old former paleontologist presents to clinic with concerns about her breast cancer risk. Her mother developed the disease in her 50s and died from it in her 60s. A younger cousin developed the disease a few years ago before the age of 50 years, but this individual was not tested for the BRCA1 and BRCA2 genes. In addition, the patient suffered from lymphoma in her 20s and had radiation to the chest. She did take hormone replacement therapy for a few years before data emerged that this may contribute to breast cancer risk. She has had several abnormal mammograms in her 50s for persistently dense breasts with subtle findings, but follow-up biopsies never showed any malignant pathology. Which of the following is true regarding magnetic resonance imaging (MRI) screening of this patient? a. No agency recommends breast MRI for a patient such as this one, who has moderately but not extraordinary risk factors for b c. Regardless of recommendations, the high sensitivity of breast MRI comes at the expense of markedly decreased specifiicity (i.e., the ability to rule out disease in healthy breasts). Page | 31 A 66-year-old female museum curator presents for a routine annual examination. On examination, a notably enlarged supraclavicular lymph node is appreciated on the right side. The lymph node is nontender and feels firm and rubbery. She denies any localized or systemic symptoms such as breast lumps, fevers, or night sweats. She has been taking conjugated estrogen tablets for 9 years since menopause, though she has not taken progestin compounds since she had a hysterectomy for heavy bleeding at age 45 years. Which of the following is true about this presentation of lymphadenopathy? a. Breast cancer always presents with axillary lymphadenopathy because the lymphatics of the breast uniformly drain into the axilla. b. Supraclavicular nodes are generally considered benign and require no further evaluation or follow-up. c. Supraclavicular nodes are found along the anterior edge of the trapezius muscle in the neck. d. Firm, e. Metastatic breast cancer cells may spread directly into the infraclavicular and then supraclavicular nodes without first causing notable changes in the axillary nodes. When is the best day to perform breast exam? 5-7 days after the first day of menstruation Consistent day of the month if no periods What are you looking for on breast exam? •Inspect for symmetry, contours, and retraction •Palpation with patient supine with arm above head •Palpate in ladder pattern, circular motion for each location CHECK INTO AXILLA Montgomery glands Sebaceous glands, produces oily protective lubricant for lactation Lactation secretion occurs with Physiological secretion with pregnancy, lactation, chest wall stimulation, sleep, and stress Galactorrhea Milky discharge unrelated to pregnancy and lactation Mammary souffle "Puff of air" heard during pregnancy and lactation (venous hum) Page | 32 Breast cancer signs Redness, peau d'orange (thickening and prominent pores), flattening instead of normal convexity, asymmetry (any drastic changes), change in nipple direction Whole breast involved typically Paget disease Occurs in breast cancer Rash, scaling, ulceration of nipple and areola Breast cancer risk factors •Increasing age •First degree family member with breast cancer •Genetic mutations (BRCA1 and BRCA2) •Personal history of precancerous breast mass •Denser breasts (confirmed by mammogram) •High dose radiation exposure •High levels estrogen hormones Recommendations for breast cancer screening Box 18-4 - page 607 Mammogram screening ages 50-74 Abdominal exam order Inspection (contours), auscultation (bowel sounds, bruits), percussion (liver size, dullness or tympany in colon), palpation (liver size, splenomegaly, masses in colon-can be from fecal matter, cancer, diverticulitis), special maneuvers (murphy-gallbladder, appendix tests) Liver will feel on inspiration Visceral abdominal pain Hollow abdominal organs forcefully contract or distention •Nonspecific, difficult to localize •Gnawing, cramping, aching •Sweating, pallor, nausea, vomiting, restlessness Somatic/parietal abdominal pain Inflammation of the parietal peritoneum •Localized or diffuse Page | 35 Prostate cancer risk factors Higher risk at age 40, increases rapidly after 50 African American Agent orange, high fat diet, obesity, cigarette smoking, cadmium exposure Screening test for prostate cancer PSA (prostate specific antigen) Proctitis s/s **Last 6 inches of colon** (not prostatisis) Anorectal pain, tenesmus, discharge, bleeding, anal fissures What is proctitis caused by IBD, STIs, trauma, bacterial infections, radiation therapy (esp for prostate cancer tx) Incarcerated hernias Contents cannot be returned to abdominal cavity Strangulation hernias Blood supply is compromised Femoral hernias have higher risk for Incarceration and strangulations How to assess for hernias With patient standing Hydrocele Localized, fluid filled lesions, encapsulated masses, transilluminates (like a cyst but deeper) Cryptorchidism Undescended testicle Occluded follicles/epidermoid cyst Dome-shaped white or yellow lesions Testicular torsion Tender, painful, scrotal swelling - EMERGENCY Page | 36 Variocele Varicose vein of spermatic cord, thick rope-like texture-usually benign Testicular cancer s/s & risk factors Painless nodules, peak incidence 15-34 years old, palpable inguinal lymph nodes, self testicular exam after warm shower Pelvic exam Always have chaperone External examination first Don't use lubricant especially near end for speculum, can alter results (use warm water) for test, not real life 30 degree downward angle Bimanual pelvic exam Adnexa includes ovaries, fallopian tubes, supporting tissue Chlamydia White discharge in men Yellow discharge in women Gonorrhea Yellow in men and women Syphilis Primary chancre-beefy red sores, secondary type: flat top lesions in women Genital warts Severely raised, can be erythematous Usually caused by HPV Herpes Smaller lesions, can be normal pigmentation, erythematous, crusted Pubic lice Severe excoriation due to itching, can sometimes see lice itself What is normal vaginal discharge Page | 1 NR509 FINAL EXAM TEST BANK WITH ACTUAL CORRECT QUESTIONS AND VERIFIED DETAILED RATIONALES ANSWERS BY EXPERTS |FREQUENTLY TESTED QUESTIONS AND SOLUTIONS | 2024 NEWEST |ALREADY GRADED A+|LATEST UPDATE A 24-year-old graphic designer presents to clinic with a concern for a breast mass. A rubbery, mobile, nontender mass is palpated in the right breast as described by the patient, which is consistent with a firbroadenoma. In describing the location of the mass, the examiner notes that it is 3 cm proximal to and 3 cm to the left of the nipple. Which of the following would be the most appropriate way to report this finding? a. "Rubbery, mobile, nontender mass located in right breast, in the 10:30 position from the nipple" b. "Rubbery, mobile, nontender mass located in right breast, in the lower outer quadrant" c. "Rubbery, mobile, nontender mass located in right breast, in the upper inner quadrant" d. "Rubbery, mobile, nontender mass located in the left breast, upper outer quadrant" e. "Rubbery, mobile, nontender mass located in right breast, in the 1:30 position from the nipple" a. "Rubbery, mobile, nontender mass located in right breast, in the 10:30 position from the nipple" A 54-year-old female dietician presents for a routine annual examination. On review of systems, she reports that she has had many breast findings over several years, including one biopsy with normal pathology. She feels that her breasts have become far less lumpy since she underwent menopause 3 years ago. Which of the following is true regarding changes in the breasts with menopause? a. Transformation of breasts to primarily fatty tissue with menopause decreases the sensitivity and specificity of mammograms. b. Estrogen in hormone replacement therapy (HRT) has no effect on breast density after menopause. c. Glandular tissue of the breast atrophies with menopause, primarily due to decrease in the number of lobules. d. Breast density has no genetic component and is entirely due to estrogen dose from endogenous and exogenous sources over the lifetime. e. Mammography performs most poorly in the menopausal and postmenopa c. Glandular tissue of the breast atrophies with menopause, primarily due to decrease in the number of lobules. An overweight 26-year-old public servant presents to the Emergency Department with 12 hours of intense abdominal pain, light-headedness, and a fainting episode that finally prompted her to seek medical attention. She has a strong family history of gallstones and is concerned about this possibility. She has not had any vomiting or diarrhea. She had a normal bowel movement this morning. Her Page | 2 βhuman chorionic gonadotropin (β-hCG) is positive at triage. She reports that her last period was 10 weeks ago. Her vital signs at triage are pulse, 118; blood pressure, 86/68; respiratory rate, 20/min; oxygen saturation, 99%; and temperature, 37.3ºC orally. The clinician performs an abdominal exam prior to her pelvic exam and, on palpation of her abdomen, finds involuntary rigidity and rebound tenderness. What is the most likely diagnosis? a. Ruptured tubal (or ectopic) pregnancy b. Acute cholecystitis c. Ruptured appendix d. P a. Ruptured tubal (or ectopic) pregnancy A 63-year-old janitor with a history of adenomatous colonic polyps presents for a well visit. Basic labs are performed to screen for diabetes mellitus and dyslipidemia. Electrolytes and liver enzymes were also measured. His labs are all normal expect for moderate elevations of aspartate aminotransferase, alanine aminotransferase, γ-glutamyl transferase, and alkaline phosphatase as well as a mildly elevated total bilirubin. He presents for a follow-up appointment and the clinician performs an abdominal exam to assess his liver. Which of the following findings would be most consistent with hepatomegaly? a. Liver span of 11 cm at the midclavicular line b. Liver span of 8 cm at the midsternal line c. Dullness to percussion over a span of 11 cm at the midclavicular line d. Dullness to percussion over a span of 8 cm at the midsternal line e. Liver palpable 3 cm below the right costal margin, mid clavicular line, on expir e. Liver palpable 3 cm below the right costal margin, mid clavicular line, on expiration A 63-year-old underweight administrative clerk with a 50-pack-year smoking history presents with a several month history of recurrent epigastric abdominal discomfort. She feels fairly well otherwise and denies any nausea, vomiting, diarrhea, or constipation. She reports that a first cousin died from a ruptured aneurysm at age 68 years. Her vital signs are pulse, 86; blood pressure, 148/92; respiratory rate, 16; oxygen saturation, 95%; and temperature, 36.2ºC. Her body mass index is 17.6. On exam, her abdominal aorta is prominent, which is concerning for an abdominal aortic aneurysm (AAA). Which of the following is her most significant risk factor for an AAA? a. Female gender b. History of smoking c. Underweight d. Family history of ruptured aneurysm e. Hypertension b. History of smoking A 76-year-old retired man with a history of prostate cancer and hypertension has been screened annually for colon cancer using high sensitivity fecal occult blood testing (FOBT). He presents for followup of his hypertension, during which the clinician scans his chart to ensure he is up to date with Page | 5 d. Seminal vesicle e. Vas deferens a. Corpora cavernosa A 29-year-old graduate student states that he is able to achieve an erection and ejaculate during sexual intercourse; however, he does not experience any pleasurable sensation of orgasm. He is otherwise healthy and is not on any medications. What is the most likely cause of his problem? a. Androgen insufficiency b. Endocrine dysfunction c. Peyronie disease d. Psychogenic e. Sexually transmitted infection (STI) d. Psychogenic Multiple processes must take place in order for a male to sustain an erection. Various cues stimulate sympathetic outflow from higher brain centers to the T11-L2 levels of the spinal cord and parasympathetic outflow from S2 to S4 reflex arcs. Local vasodilatation within the penis erectile tissue results from increased levels of which of the following? a. Follicle-stimulating hormone (FSH) b. Gonadotropin-releasing hormone (GRH) c. Luteinizing hormone (LH) d. Nitric oxide (NO) and cyclic guanosine monophosphate (cGMP) e. Testosterone d. Nitric oxide (NO) and cyclic guanosine monophosphate (cGMP) The human papillomavirus (HPV) can cause genital warts in males and females as well as cervical cancer in females. Vaccination against HPV is available and should be offered to males between what ages? a. 6-9 months b. 1-3 years c. 5-7 years d. 9-21 years e. 30-50 years d. 9-21 years A 32-year-old male complains of a painless, cystic mass just above his left testicle. During the physical examination, a strong flashlight is placed behind the scrotum through the area in question and transillumination is noted. What is the most likely diagnosis? a. Direct hernia b. Indirect hernia c. Spermatocele Page | 6 d. Testicular tumor e. Varicocele c. Spermatocele A 25-year-old graduate student presents to the clinic complaining of scrotal pain, which has been increasing over the past 2 days. He is sexually active and has had unprotected intercourse with multiple partners in the past couple of weeks. On examination, some mild to moderate swelling of the scrotum on the right and tenderness with palpation of the right testicle are notes. What is the most likely diagnosis? a. Acute epididymitis b. Hydrocele c. Primary syphilis d. Spermatocele e. Testicular cancer a. Acute epididymitis A 32-year-old elementary teacher requests a workup for infertility. He and his wife have been trying to conceive for the last 2 years. He reports that his wife has been evaluated and does not appear to have any infertility issues. The overall examination does not reveal any significant abnormalities. He is of average height and weight and has normal secondary sex characteristics of the genitalia. Of the following, which would be most likely be abnormal and causing male infertility? a. 5α-Dihydrotestosterone b. 5α-Reductase c. Follicle-stimulating hormone (FSH) d. Luteinizing hormone (LH) e. Thyroid-stimulating hormone (TSH) c. Follicle-stimulating hormone (FSH) While performing a physical examination on male patients, it is possible to palpate multiple structures in relation to the inguinal canal and related hernias. Which of the following is not palpable during an external examination of the abdominal wall or inguinal region? a. External inguinal ring b. Internal inguinal ring c. Pubic tubercle d. Anterior superior iliac spine e. Direct inguinal hernia b. Internal inguinal ring A 20-year-old college student presents for his annual physical examination. He recently became sexually active and is inquiring about the best means of preventing sexually transmitted infections (STIs). Of the following, which would be the most effective means of prevention? Page | 7 a. Early withdrawal b. Male condoms c. Spermicides d. Diaphragms e. Cervical caps b. Male condoms A 21-year-old college student presents to the student health clinic for a full physical examination. He is generally healthy; however, he reports that he has had sexual intercourse with multiple partners in the past couple of months. He noticed a small lesion on the shaft of his penis a few days ago. While performing the examination, he unwillingly achieves an erection. How should the clinician proceed at this point? a. Stop the examination immediately. b. Have him return to see another provider. c. Explain this is a normal response and finish the examination. d. Tell him the examination cannot proceed until the erection subsides. e. Assume that he is malingering. c. Explain this is a normal response and finish the examination. A 45-year-old driver's education instructor presents to the clinic for heavy periods and pelvic pain during her menses. She reached menarche at age 13 years and has had regular periods except during her pregnancies. She is a G4P3013 and does not use birth control as her husband has had a vasectomy. She states this has been going on for about a year but seems to be getting worse. Her last period was 1 week ago. On bimanual exam, a large midline mass halfway to the umbilicus is palpated. Each adnexal area is nonpalpable. Her rectal exam is normal. Her body mass index (BMI) is 27. What is the best explanation for her physical finding? a. Large colonic stool b. Ovarian mass c. Fibroids d. 4-Month pregnancy e. Bartholin gland enlargement c. Fibroids A 32-year-old G0 woman comes for evaluation on why she and her husband have been unable to get pregnant. Her husband has been married before and has two other children, ages 7 and 4 years. The patient relates she began her periods at age 12 and has been fairly regular ever since. She began oral contraceptive pills from when she got married until last year, when she began to try for a pregnancy. Before this she had regular cycles for 10 years. She has had a history of five prior partners. She relates she was once treated for a severe genital infection when she was in college. Based on this patient's history, what is the best explanation for her infertility? Page | 10 c. Withdraw the speculum slightly and reposition it on a different slope A 63-year-old office worker comes to the clinic for her women's health exam. Her last Pap smear was 5 years ago and was normal. She is married and has been with the same sexual partner for the last 35 years. After performing the majority of the exam, the clinician decides to do a speculum exam to collect cytology for Pap smear. What is the correct position to have the patient in for her speculum exam? a. Sitting b. Supine c. Prone d. Trendelenburg e. Lithotomy e. Lithotomy A 68-year-old retired patient presents to the clinic complaining about feeling like something is falling out of her vagina. She is a G6P6007 and had all her children vaginally, even the twins. She went through menopause at age 55 years, and, for the last few months, she has felt this falling sensation. On exam, an anterior bulge in the vaginal wall is apparent when she bears down. Weakness in which muscle would best account for the anterior bulge in the vaginal wall? a. Levatori ani b. Anal sphincter c. Pubis symphysis d. Ischiocavernosus muscle e. Bulbocavernosus muscle a. Levatori ani A 49-year-old male nurse experiences fecal incontinence after a motor vehicle accident that left him paralyzed below the waist. He asks his rehabilitation physician about the control of this function in a person without his injuries. Which of the following is true regarding the muscle control of the anal sphincter? a. The internal anal sphincter is under voluntary control, whereas the external anal sphincter is under involuntary control. b. The internal anal sphincter is under involuntary control, whereas the external anal sphincter is under voluntary control. c. Both internal and external anal sphincter are under voluntary control. d. Both internal and external anal sphincter are under involuntary control. e. Control of the anal sphincters is variable between individuals. b. The internal anal sphincter is under involuntary control, whereas the external anal sphincter is under voluntary control. A 62-year-old male who is undergoing evaluation for possible prostate cancer strongly declines a rectal examination, stating that, "Some trainee once did that and it hurt badly." Which of the following is true Page | 11 about the innervation of the anus and rectum that may explain this patient's experience of discomfort? a. The rectum contains primarily somatic nerves, whereas the anal canal contains primarily visceral nerves, making the rectum the most likely source of this patient's discomfort. b. The rectum contains primarily somatic nerves, whereas the anal canal contains primarily visceral nerves, making the anus the most likely source of this patient's discomfort. c. Proximal to the dentate line, the lower gastrointestinal tract is innervated primarily by somatosensory nerves, making the proximal reach of the examination the most likely site of this patient's pain d. The anal canal has a rich somatosensory innervation, d. The anal canal has a rich somatosensory innervation, making poorly directed examinations painful in this area. A 54-year-old male with a strong family history of breast and prostate cancer presents to his primary care provider to discuss prostate screening. His father died at age 73 years from prostate cancer that was not detected on routine digital rectal examinations (DREs), and he would like to minimize his chance of a similar occurrence. Which of the following is true regarding the anatomy and screening of the prostate by DRE? a. All three lobes of the prostate are palpable on DRE. b. The seminal vesicles are palpable distal to the prostate on DRE. c. The median lobe of the prostate is located anterior to the urethra and is not palpable on DRE. d. The median sulcus divides the lateral lobes from the median lobe and is palpable on DRE. e. A prostate of 5 cm diameter without palpable nodes or masses represents a normal prostate examination. c. The median lobe of the prostate is located anterior to the urethra and is not palpable on DRE. A third-year medical student rotating on the internal medicine service performs a digital rectal examination (DRE) on a 56-year-old female patient. The patient has been admitted for suspicion of a myocardial infarction, and confirmation that there is no blood in the stool is required before anticoagulation can be started. The student reports that the fecal occult blood test was negative but notes that he palpated a structure through the anterior rectum that he could not identify. The attending physician confirms normal anatomy and reviews with the student that the most likely identity of the structure palpable is which of the following a. Sacrum b. Pectinate line c. Uterine fundus d. Prostat e. Cervix e. Cervix A 45-year-old female executive reports to her primary care provider that she has recently experienced a change in the patterns of her bowel movements. She expresses a great concern as her family history includes a maternal aunt who died of colon cancer at age 49 years; her mother has had colonoscopies Page | 12 every 3 years with numerous adenomatous polyps removed. Which of the following historical elements would be the most concerning for colon cancer in this patient? a. Long-term history of hemorrhoids b. Recent history of black, tarry stools c. Remote history of anal pruritus d. New-onset anal fissure e. Recent onset of small-caliber stools e. Recent onset of small-caliber stools A 49-year-old customer service representative presents to his gastroenterologist for follow-up of his long-standing inflammatory bowel disease (IBD). He was diagnosed with ulcerative colitis (UC) at age 37 years and has had irregular care for this condition since then. His sole colonoscopy was done at the time of diagnosis 12 years ago. His only relevant family history is of prostate cancer in his father; his mother and sisters are healthy. Which of the following is true about recommended screening for colon cancer in this patient? a. The patient should begin screening for colon cancer 10 years prior to the age of onset of his father's prostate cancer. b. The patient should undergo colonoscopy for his bowel condition, which confers risk of colon cancer. c. The patient is due for routine age-based colon cancer screening by colonoscopy regardless of his risk factors. d. The patient has a reassuring family history and b. The patient should undergo colonoscopy for his bowel condition, which confers risk of colon cancer. A 49-year-old male with well-controlled HIV undergoes a proctoscopic examination as routine screening for anal cancer. The patient is asymptomatic and specifically denies complaints of frequent urination (frequency), large volume of urination (polyuria), or repeated urination at night (nocturia). Under direct visualization, the clinician observes a clear, circumferential demarcation of proximal versus distal tissue. This demarcation was not palpable on digital rectal examination (DRE) prior to proctoscopy. What is the most likely origin of this finding? a. Pathological constriction of the anal canal b. Normal anatomy of the mucosal surface c. Carcinoma d. Valve of Houston e. External anal sphincter b. Normal anatomy of the mucosal surface A 34-year-old female reports anal pain with defecation. She notes incidentally to this complaint that she has developed episodic abdominal discomfort and sores in her mouth. Anoscopic examination reveals anal fissures that appear to be her source of pain. Which of the following underlying conditions is the clinician most likely to find? Page | 15 c. Compare the strength of his trapezia muscles d. Assess his ability to extend his back e. Check for listing of his trunk c. Compare the strength of his trapezia muscles During a musculoskeletal examination, the clinician instructs the patient to look over one shoulder, and then the other shoulder. This action assesses the movement of which muscle(s)? a. Scalenes b. Sternocleidomastoid (SCM) c. Splenius capitis d. Prevertebral muscles e. Splenius cervicis b. Sternocleidomastoid (SCM) During a musculoskeletal examination of the spine, what is the action(s) of the erector spinae muscle group? a. Rotation of the spine b. Extension of the spine c. Flexion of the spine d. Lateral bending of the spin e. Rotation and lateral bending of the spine b. Extension of the spine The clinician is seeing a middle-aged patient who has a diagnosis of lumbar spinal stenosis. The patient's history is consistent with this diagnosis as he has pain in the back with walking that improves with rest. Which physical sign(s) are most consistent with his diagnosis? a. Hyperreflexia of the lower limb b. Pelvic tilt or drop c. Thoracic kyphosis d. Positive straight-leg raise e. Flexed forward posture with lower extremity weakness e. Flexed forward posture with lower extremity weakness The clinician is seeing a 58-year-old patient with a diagnosis of arthritis. The patient complains of pain in his knees, hips, hands, wrists, neck, and low back. Based on which joints are involved, the patient most likely has which joint problem? a. Osteoarthritis (OA) b. Rheumatoid arthritis (RA) c. Psoriatic arthritis Page | 16 d. Gout e. Polymyalgia rheumatica a. Osteoarthritis (OA) A 62-year-old patient with rheumatoid arthritis (RA) complains of increased joint stiffness. What characteristic(s) are consistent with her diagnosis of RA? a. Stiffness follows joint activity. b. Joint distribution is asymmetrical. c. Swelling of the synovial tissue is seen in joints and tendon sheaths. d. It most frequently involves the first metatarsophalangeal joint. e. Tophi are found in the subcutaneous tissue. c. Swelling of the synovial tissue is seen in joints and tendon sheaths. A 55-year-old woman with a headache explains to the clinician that she has had headaches before, but this one is unusual because of some new symptoms. Which of the following symptoms would prompt an immediate investigation? a. The headache is similar in nature to prior ones she has had for decades but more severe. b. The patient had a car accident and minor head trauma about 3 months ago. c. The patient also has developed fever and night sweats and thinks she lost some weight. d. The headache comes and goes. e. The patient lost her glasses. c. The patient also has developed fever and night sweats and thinks she lost some weight. In the case of a middle-aged female with a pounding headache, what is an effective question to ask the patient? a. Does the patient have any aura prior to the headaches? b. How old is the patient? c. Is she feeling stressed? d. Does she think she is losing her memory? e. Has she ever seen anyone with a stroke? a. Does the patient have any aura prior to the headaches? A 35-year-old female patient has had migraines for much of her adult life. Ather regular checkup, she is healthy, takes no medications except oral contraceptive pills (OCPs), exercises, and has a steady job. Her only complaint is that her migraines seem to have become worse, and, for the past few weeks, she has been waking up at night with headache and also nausea. Which of the following is the best course of action? a. Reassure her that this is a common pattern with migraines. b. Order studies to evaluate potential transient ischemic attacks (TIAs) because she is on OCPs. Page | 17 c. Take a further history and perform a very careful neurological examination. d. Treat her for sinusitis. e. Prescribe a strong medication for her migraines. c. Take a further history and perform a very careful neurological examination. d. Treat her for sinusitis. An 82-year-old grandmother presents to the Emergency Department in the care of her extended family with new-onset speech impairment. According to family members, the patient awoke with this symptom as well as difficulty in understanding questions or following commands. Her past medical history is remarkable for atrial fibrillation but no other notable conditions. On examination, her speech is verbose but poorly comprehensible and lacks meaning. She is unable to follow simple commands. Which of the following best describes her speech disorder? a. Dysphonia with expressive deficit b. Dysarthria c. Wernicke aphasia d. Global aphasia e. Broca aphasia c. Wernicke aphasia A 74-year-old bus driver is delivered to the hospital via emergency transport after an astute passenger noted that the patient exhibited drooping facial features and slurred speech. The patient was diagnosed rapidly with ischemic (nonhemorrhagic) stroke, and urgent intervention lead to a near complete recovery from his symptoms. The astute passenger was thanked and congratulated for recognizing the signs of acute stroke; this individual credited this recognition to a public safety awareness campaign that outlined the critical public health need to recognize strokes early. Which of the following statements is true for risks and rapid recognition of suspected strokes? a. Atrial fibrillation is not a risk factor for ischemic stroke in individuals age ≥75 years. b. Hypertension is the leading risk factor for both ischemic and hemorrhagic stroke. c. Obesity with normal glucose tolerance is not a risk factor for stroke. b. Hypertension is the leading risk factor for both ischemic and hemorrhagic stroke. A 70-year-old male presents to the Emergency Department accompanied by his wife, who is concerned that he has experienced a stroke. She states that he awoke with drooping of the right side of his mouth. He has a history of hypertension and diet-controlled diabetes, but no history of prior transient ischemic attacks (TIAs), strokes, or neurologic deficits. Physical examination reveals a wellnourished, right-handed male, who has an obvious flattening of the right nasolabial fold at rest. He is unable to close his right eye, wrinkle his forehead, or raise his eyebrows. The remainder of the neurologic examination is symmetric with intact strength and normal deep tendon reflexes. Based on this history and physical examination, which of the following statements is true? a. The patient most likely has a central upper motor neuron lesion involving cranial nerve (CN) VII (the facial nerve). b. The patient most likely has a Page | 20 b. Brainstem c. Lumbar spinal cord d. Peripheral nerve e. Distal muscle d. Peripheral nerve A mother brings her 15-month-old toddler to the clinic for his preventive health care visit. The clinician takes the history and observes the child's interactions and behaviors and is then ready to begin the rest of the examination. Which of the following best describes the general approach to the pediatric examination of the young child? a. Always give immunizations prior to beginning the examination. b. Examine the child in the same order as for an adult patient. c. Children age <2 years do not need to be examined. d. Begin with least invasive parts of the examination first. e. Never examine a young child in the mother's lap. d. Begin with least invasive parts of the examination first. In caring for children, physicians and other clinicians need to understand child development. Of the following, which is a principle of normal child development? a. Child development proceeds along a predictable pathway in a healthy child. b. There is minimal variation in when children achieve milestones. c. All delays in development can be explained by one or two risk factors. d. Regression in developmental skills is not a cause for concern. e. A child's developmental level can be ignored in conducting an examination. a. Child development proceeds along a predictable pathway in a healthy child. The parents of a 21-month-old child explain that their son used to speak nearly 50 words and was using 2-word phrases. In the last month or so, the child has not been using as many words and tends to echo what is being said to him rather than use language spontaneously. They want to know if this is normal. After taking a thorough developmental history, the clinician finds that the child makes poor eye contact and does not play with toys in a purposeful manner. The physical examination is normal except for the child's limited social interactions. There is a family history of autism in two first cousins. Which of the following would be the best response to the parents at this time? a. Reassure the parents that all toddlers lose skills at some point in development. b. Reassure the parents that the child is fine as long as he has not lost skills in other domains. c. Send the child to the Emergency Department (ED). d. Re d. Refer the child to a developmental and behavioral pediatrician. A clinician is meeting the mother of a 5-year-old with asthma for the first time. The mother notes that the asthma has been poorly controlled and that the child has had multiple hospitalizations. The clinician Page | 21 inquires about family stressors and finds that the parents are divorced, the mother recently lost her job, and the child spent 2 months living with her grandparents who both smoke. Which of the following is the best example of the role of health promotion with this family? a. Plan less frequent pediatric visits because the family will take too much time. b. Delay immunizations because of the family stressors. c. Postpone vision and hearing screening because the child may not pass d. Reassure the parent that the family stressors are not impacting the child's asthma. e. Develop a health promotion plan that includes more frequent visits and guidance to assist family with stressors and improve the child's asthma s e. Develop a health promotion plan that includes more frequent visits and guidance to assist family with stressors and improve the child's asthma symptoms. A clinician is reading the chart of a full-term newborn whose mother had an uneventful pregnancy in the hospital for the first time on the day of birth. In reviewing the infant's chart, the clinician notes that, in the delivery room, at 5 minutes, the infant had a heart rate >100, strong respiratory effort, was crying vigorously, moving actively, and had good color except for some acrocyanosis of the hands and feet. This infant's APGAR score is closest to which of the following? a. 9 b. 7 c. 5 d. 3 e. 1 a. 9 A newborn who is floppy and limp, blue in color, with a heart rate of 60, and minimal respiratory effort has just been delivered. The infant has no grimace and only a very weak cry. What is the best immediate response to the infant in this situation? a. Discuss the infant's poor appearance with the parents who are both in the room. b. Suction the infant's mouth while waiting to calculate the 5-minute APGAR score. c. Dry the infant off and swaddle him. d. Begin neonatal resuscitation. e. Order a chest x-ray. d. Begin neonatal resuscitation. An infant is born 4 weeks preterm to a mother with a history of hypertension, severe diabetes, and alcohol abuse. The infant is noted to be small for gestational age (SGA), weighing just 1,500 g. Which of the following is the most important reason for assessing both gestational age and birth weight for any infant? a. These two factors help to anticipate certain medical and developmental problems. b. The parents should be informed of these. Page | 22 c. A SGA infant is at low risk for neonatal problems. d. A premature infant with a weight appropriate for gestational age has a very low risk for neonatal problems. e. Full-term, appropriate-for-gestational age (AGA) infants having a high risk of long-term problems. a. These two factors help to anticipate certain medical and developmental problems. A clinician arrives at the hospital several hours after the birth of a full-term infant. The infant is rooming in with her parents and appears to be doing well. There were no problems with the pregnancy, labor, or delivery. The nurse asks if the baby should be taken back to the nursery for examination. What is the best response to the nurse? a. State that it will be much more efficient to conduct the examination in the nursery. b. Note that the infant already had an examination in the delivery room and does not need another examination so soon. c. State that the infant should be examined in the presence of the parents so they can be taught about what their newborn can do. d. Note that the lighting is better in the newborn nursery. e. Refer the parents to a good book on newborns and wheel the infant back to the newborn nursery to conduct the examination. c. State that the infant should be examined in the presence of the parents so they can be taught about what their newborn can do. The nurse in the newborn nursery reports that she is concerned about Baby Boy Jones, who was born full-term by cesarean section for failure to progress. The pregnancy was complicated only by a maternal urinary tract infection in the first trimester. He had APGARs of 9 and 10 at 1 and 5 minutes, respectively, and had been doing well. However, now, on the fourth day of life, the infant has developed a tremor. Which of the following factors would cause the most concern about the tremor? a. There is a history of benign tremor in elderly family members. b. The infant also has asymmetric limb movements. c. The infant's vital signs are normal. d. The tremor is brief and only present when the infant is crying vigorously. e. The infant lies in a symmetric position with limbs flexed when relaxed. b. The infant also has asymmetric limb movements. A mother brings her 9-month-old son to the practice for the first time, concerned that he is not yet sitting by himself. After taking a careful history, the physician notes that the infant has good head control and can grasp a rattle but is unable to roll over, crawl, or pull to stand. What should the clinician explain to the mother? a. Her child is progressing normally and does not need further evaluation. b. Gross motor development proceeds from peripheral skills, such as finger feeding, to central skills, such as sitting. c. As long as the child is babbling, delays in gross motor skills are not a concern. Page | 25 vaccination should be given postpartum to protect future pregnancies from the effects of congenital rubella. b. Hepatitis B, measles/mumps/rubella (MMR), and influenza vaccines are safe during pregnancy. c. Polio and influenza vaccinations are not safe during pregnancy and should never be utilized. d. RhoGAM is a vaccine specific to pregnancy that should be given to all pregnant women e. No vaccines are safe during pregnancy, and the risks of vaccination outweigh the benefits of immunity to infectious diseases. a. If a pregnant woman does not show sufficient titers to rubella, measles/mumps/rubella (MMR) vaccination should be given postpartum to protect future pregnancies from the effects of congenital rubella. A 31-year-old marathon runner presents for prenatal care with her first pregnancy. She is in her second trimester and is experiencing some fatigue and muscle aches. Her prepregnancy body mass index (BMI) was noted at 19.2. How should she be counseled on exercise and nutrition during pregnancy? a. She should gain at least 40 pounds during the pregnancy to account for being underweight at the time of conception. b. Immersion in hot water is a safe and effective nonmedicinal way of coping with musculoskeletal complaints during pregnancy. c. She should increase her calorie intake to 300 calories per day or more from her prepregnancy baseline. d. She should avoid unpasteurized dairy products and delicatessen meats due to the risk of mycobacteria, shigellosis, and brucellosis. e. She should switch from running to weight-lifting (e.g., bench press) to maintain muscle mass while avoiding the stressors of running on the fetu c. She should increase her calorie intake to 300 calories per day or more from her prepregnancy baseline. A 29-year-old G2P1 presents to the clinic after a positive home pregnancy test. She confides at the appointment that her male partner has become increasingly abusive lately and once struck her while she was holding her older child. How should she be counseled? a. Ask that she bring the partner to all appointments so that he can be included in decisions and thus feel less threatened and less likely to harm the patient again. b. Reassure her that no matter what she reveals, all information she discloses will be kept strictly confidential. c. Ask open-ended questions, allow her to make decisions that she feels are best for herself given the circumstance, and provide immediate or long-term referrals to domestic violence resources. d. Reassure her that she is safe as very few pregnant women are hurt or murdered by their partners, who generally become less violent during the vulnerable period of pregnancy e. Demand that s c. Ask open-ended questions, allow her to make decisions that she feels are best for herself given the circumstance, and provide immediate or long-term referrals to domestic violence resources. Page | 26 A 34-year-old G3P2 at 27 weeks' gestation is referred to the clinic upon discharge from a correctional institution where she has been incarcerated for 25 days for a drug offense. She denies any further substance abuse, but her behavior is concerning for intoxication, and she smells of alcohol and cigarettes. The clinician inquires about her drug use with open-ended questions and counsels her that which of the following is true? a. Pregnant women are not routinely screened for hepatitis C, but this test should be added to the panel of prenatal blood tests for patients with a history of intravenous drug use. b. Cigarettes are a rare cause of low birth weight in the growing fetus. c. Women can safely drink one alcoholic drink per day without risk of fetal alcohol syndrome. d. If a pregnant patient does not intend to quit tobacco, she should not bother to cut down as there is no benefit to the pregnancy from decreased a. Pregnant women are not routinely screened for hepatitis C, but this test should be added to the panel of prenatal blood tests for patients with a history of intravenous drug use. A 78-year-old woman presents to clinic with her two daughters, who are concerned about her continued ability to live independently. She has thus far been highly self-reliant and is opposed to the idea of leaving of her home of 30 years. The clinician performs a complete history and physical exam (including mental status and memory testing) as well as orders laboratory tests before providing the patient and her family the finding that she has age-appropriate changes that do not reflect any particular disease process. Which of the following findings is most consistent with the normal aging process and does not impair the ability to live alone? a. Age-related cognitive decline b. Mild cognitive impairment c. Decreased level of thyroid hormone d. Persistent urinary incontinence e. Decreased adipose-to-muscle ratio a. Age-related cognitive decline Which of the following best describes the role of the health practitioner in caring for the aging American population? a. Prepare all persons age ≥65 years for the eventuality that they will become frail. b. Understand that the older population is generally homogenous with little variation in needs. c. Evaluate geriatric conditions in terms of functionality and quality of life rather than via traditional disease models. d. Employ the same disease models used to treat younger patients with chronic disease. e. Assure that all elders complete an annual physical examination. c. Evaluate geriatric conditions in terms of functionality and quality of life rather than via traditional disease models. Which of the following is true about hair in the aging adult? Page | 27 a. Age-related hair loss in males is normal only after age 50 years. b. Age-related hair loss on the scalp is abnormal in women and should be evaluated to rule out underlying pathology. c. Although hair loss occurs in both sexes, hair on the head, trunk, legs, and pubic hair is invariably spared any age-related changes. d. Women may experience the development of sparse coarse facial hair in their mid-50s. e. Age-related hair changes are the same for all individuals regardless of ethnicity or race. d. Women may experience the development of sparse coarse facial hair in their mid-50s. A 75-year-old female in generally good health presents to a new primary care provider after she recently moved to a new city. She takes no prescribed medications, but she has been told in the past that her blood pressure was borderline elevated and might require treatment at some time in the future. Which of the following findings during the physical examine is consistent with the normal aging process and not a sign of cardiovascular disease? a. A narrowed pulse pressure with increased systolic and diastolic components b. A widened pulse pressure with increased systolic pressure (up to 140) and decreased diastolic pressure c. An unchanged pulse pressure with equal increases in both systolic and diastolic pressures d. A drop in systolic pressure of 25 mm Hg when rising from a supine to standing position e. An isolated increase in systolic blood pressure to >150 mm Hg b. A widened pulse pressure with increased systolic pressure (up to 140) and decreased diastolic pressure A 66-year-old recently retired restaurant worker presents to his primary care provider with a concern about hearing loss. He relates a history of difficulty distinguishing voices in crowded settings when significant background noise exists, which hastened his retirement. Which of the following is true about this patient's experience with hearing and the aging process? a. His experience is consistent with the normal aging process. b. Any hearing impairment that causes functional decline warrants formal testing and evaluation. c. Early age-related hearing loss initially affects lower-pitched sounds. d. Decreased hearing acuity associated with aging is formally known as hypoacusis. e. Age-related declines in hearing does not begin until age ≥75 years. b. Any hearing impairment that causes functional decline warrants formal testing and evaluation. Medications carry both risks and benefits for older patients. Although the risks of polypharmacy (the use of many medications at once) are very well known, many older patients take many medications for a variety of conditions. Which of the following best describes medication prescribing and utilization in the older adult population? a. Although older patients take more medications than younger adults, their rate of hospitalization for drug-related adverse reactions is the same. b. Older patients rarely take or have adverse effects from sleep medications. Page | 30 e. Prolactinoma A 22-year-old G0P0 undergraduate student presents to clinic after finding a breast mass on breast self- examination (BSE) at home. The mass is nontender without skin changes, erythema, or overlying swelling. She has heard that most breast cancers are found by patients themselves, and she is very concerned that she may have breast cancer. Which of the following is true about BSE and self-detection of breast cancer? a. Most masses that women find at home and bring to a provider's attention turn out to be malignant. b. This patient is more likely to find a fibroadenoma than a cancer on self-examination. c. The most likely breast mass this patient is likely to find in herself is an abscess complicating underlying mastitis. d. Because of this patient's age, breast masses should not be pursued with imaging and diagnosis because the risk of cancer is so low. e. BSE is universally recommended because of very high sensitivity b. This patient is more likely to find a fibroadenoma than a cancer on self-examination. A 48-year-old female psychologist presents to clinic with concerns about her breast cancer risk after an age-matched cousin was recently diagnosed with this disease. This cousin is the third family member on her father's side in as many years to be diagnosed with breast cancer, including the patient's own father, who had surgery and subsequent treatment 3 years ago for breast cancer. The patient has little other knowledge of her family history, only that her grandparents independently arrived from Eastern Europe near the end of World War II and were among very few members of their family that survived the war. The patient has read about testing for the breast cancer genes (BRCA1 and BRCA2) and desires further information about whether this would be appropriate for her. Which of the following is true about this patient's indications for BRCA testing? a. Her familial lineage is irrelevant to her risk of BRCA genes and d. This patient carries several risk factors that together justify BRCA testing. A 68-year-old former paleontologist presents to clinic with concerns about her breast cancer risk. Her mother developed the disease in her 50s and died from it in her 60s. A younger cousin developed the disease a few years ago before the age of 50 years, but this individual was not tested for the BRCA1 and BRCA2 genes. In addition, the patient suffered from lymphoma in her 20s and had radiation to the chest. She did take hormone replacement therapy for a few years before data emerged that this may contribute to breast cancer risk. She has had several abnormal mammograms in her 50s for persistently dense breasts with subtle findings, but follow-up biopsies never showed any malignant pathology. Which of the following is true regarding magnetic resonance imaging (MRI) screening of this patient? a. No agency recommends breast MRI for a patient such as this one, who has moderately but not extraordinary risk factors for b c. Regardless of recommendations, the high sensitivity of breast MRI comes at the expense of markedly decreased specifiicity (i.e., the ability to rule out disease in healthy breasts). Page | 31 A 66-year-old female museum curator presents for a routine annual examination. On examination, a notably enlarged supraclavicular lymph node is appreciated on the right side. The lymph node is nontender and feels firm and rubbery. She denies any localized or systemic symptoms such as breast lumps, fevers, or night sweats. She has been taking conjugated estrogen tablets for 9 years since menopause, though she has not taken progestin compounds since she had a hysterectomy for heavy bleeding at age 45 years. Which of the following is true about this presentation of lymphadenopathy? a. Breast cancer always presents with axillary lymphadenopathy because the lymphatics of the breast uniformly drain into the axilla. b. Supraclavicular nodes are generally considered benign and require no further evaluation or follow-up. c. Supraclavicular nodes are found along the anterior edge of the trapezius muscle in the neck. d. Firm, e. Metastatic breast cancer cells may spread directly into the infraclavicular and then supraclavicular nodes without first causing notable changes in the axillary nodes. When is the best day to perform breast exam? 5-7 days after the first day of menstruation Consistent day of the month if no periods What are you looking for on breast exam? •Inspect for symmetry, contours, and retraction •Palpation with patient supine with arm above head •Palpate in ladder pattern, circular motion for each location CHECK INTO AXILLA Montgomery glands Sebaceous glands, produces oily protective lubricant for lactation Lactation secretion occurs with Physiological secretion with pregnancy, lactation, chest wall stimulation, sleep, and stress Galactorrhea Milky discharge unrelated to pregnancy and lactation Mammary souffle "Puff of air" heard during pregnancy and lactation (venous hum) Page | 32 Breast cancer signs Redness, peau d'orange (thickening and prominent pores), flattening instead of normal convexity, asymmetry (any drastic changes), change in nipple direction Whole breast involved typically Paget disease Occurs in breast cancer Rash, scaling, ulceration of nipple and areola Breast cancer risk factors •Increasing age •First degree family member with breast cancer •Genetic mutations (BRCA1 and BRCA2) •Personal history of precancerous breast mass •Denser breasts (confirmed by mammogram) •High dose radiation exposure •High levels estrogen hormones Recommendations for breast cancer screening Box 18-4 - page 607 Mammogram screening ages 50-74 Abdominal exam order Inspection (contours), auscultation (bowel sounds, bruits), percussion (liver size, dullness or tympany in colon), palpation (liver size, splenomegaly, masses in colon-can be from fecal matter, cancer, diverticulitis), special maneuvers (murphy-gallbladder, appendix tests) Liver will feel on inspiration Visceral abdominal pain Hollow abdominal organs forcefully contract or distention •Nonspecific, difficult to localize •Gnawing, cramping, aching •Sweating, pallor, nausea, vomiting, restlessness Somatic/parietal abdominal pain Inflammation of the parietal peritoneum •Localized or diffuse Page | 35 Prostate cancer risk factors Higher risk at age 40, increases rapidly after 50 African American Agent orange, high fat diet, obesity, cigarette smoking, cadmium exposure Screening test for prostate cancer PSA (prostate specific antigen) Proctitis s/s **Last 6 inches of colon** (not prostatisis) Anorectal pain, tenesmus, discharge, bleeding, anal fissures What is proctitis caused by IBD, STIs, trauma, bacterial infections, radiation therapy (esp for prostate cancer tx) Incarcerated hernias Contents cannot be returned to abdominal cavity Strangulation hernias Blood supply is compromised Femoral hernias have higher risk for Incarceration and strangulations How to assess for hernias With patient standing Hydrocele Localized, fluid filled lesions, encapsulated masses, transilluminates (like a cyst but deeper) Cryptorchidism Undescended testicle Occluded follicles/epidermoid cyst Dome-shaped white or yellow lesions Testicular torsion Tender, painful, scrotal swelling - EMERGENCY Page | 36 Variocele Varicose vein of spermatic cord, thick rope-like texture-usually benign Testicular cancer s/s & risk factors Painless nodules, peak incidence 15-34 years old, palpable inguinal lymph nodes, self testicular exam after warm shower Pelvic exam Always have chaperone External examination first Don't use lubricant especially near end for speculum, can alter results (use warm water) for test, not real life 30 degree downward angle Bimanual pelvic exam Adnexa includes ovaries, fallopian tubes, supporting tissue Chlamydia White discharge in men Yellow discharge in women Gonorrhea Yellow in men and women Syphilis Primary chancre-beefy red sores, secondary type: flat top lesions in women Genital warts Severely raised, can be erythematous Usually caused by HPV Herpes Smaller lesions, can be normal pigmentation, erythematous, crusted Pubic lice Severe excoriation due to itching, can sometimes see lice itself What is normal vaginal discharge Page | 37 Clear, white, may have white clumps, not odorous Candidiasis Thick, white, curdy discharge, not odorous, severe itching, vaginal soreness, pain with urination and intercourse (dyspareunia) BV Gray, white, thin discharge, very odorous, worse after intercourse caused by alterations in pH flora of vagina vagina = 4 sperm = 7.5 why it is worse after intercourse Myomas Benign uterine tumors Can be mistaken for ovarian mass if projecting laterally Uterine tumors can be referred to as Fibroids Pelvic inflammatory disease caused by Most common: STD (chlamydia/gonorrhea), ectopic pregnancy, recent IUD insertion Cervical cancer Extensive cauliflower like growth-confirm with PAP and colposcopy Risk factors of cervical cancer Immunosuppression, long term oral contraceptive use, Chlamydia infection, precancerous lesions, in utero exposure to Diethylstilbestrol (DES), more than 3 full term pregnancies Increased risk: MOST COMMON HPV Other increased risk with early age intercourse, frequent intercourse, multiple partners, smoking MS assessment IPROMS (inspection, palpation, ROM, special maneuvers), Articular (altered active and passive ROM) Non-articular (altered active ROM) Page | 40 Masses Tenderness Liver edge Spleen What two conditions require special maneuvers when assessing abdomen? Appendicitis (McBurney Point tenderness) Cholecystitis (Positive Murphy's) What causes visceral pain? Hollow organs (intestine or biliary tree) forcefully contract and become distended or stretched Solid organs (liver/spleen) can generate same pain when they swell against their capsules [alcoholic hepatitis] Visceral periumbilical in early acute appendicitis What does visceral pain feel like? Difficult to localize Gnawing, cramping, aching Associated symptoms: sweating, pallor, N/V, restless What causes somatic pain? What does it feel like? Pain caused by inflammation of the parietal peritoneum localized or diffuse steady aching more severe than visceral aggravated by movement What does GERD feel like? rising retrosternal burning pain may occur with coughing What makes GERD worse? alcohol chocolate citrus peppermint coffee bending over or lying after eating What are 2 common causes of peptic ulcer disease? Page | 41 H. Pylori NSAIDs Gastric vs duodenal ulcers gastric ulcer pain right after eating duodenal ulcer pain occurs hours after eating Both: GERD hematemesis melena Treatment for PUD Stop irritating factors (Alcohol, NSAIDs, smoking) PPIs, H2RA, antacids Abx for treatment of H.pylori 6 maneuvers for appendicitis McBurney Point tenderness on direct palpation - slowly apply pressure and quickly release to see if rebound tenderness Rovsing - RLQ rebound tenderness when LLQ palpated Psoas - Raise thigh (flexion) to contract psoas muscle (pain = positive sign) Obturator - Flex right thigh at hip, bend knee and internally rotate to contract obturator muscle Rectal exam - right-sided rental tenderness Pelvic exam - palpable inflamed appendix Symptoms associated with appendicitis (besides pain) fever increased WBC N/V bowel changes Diverticulosis vs diverticulitis Diverticulosis occurs when small, bulging pouches (diverticula) develop in your digestive tract. When one or more of these pouches become inflamed or infected, the condition is called diverticulitis. Symptoms of diverticulitis LLQ pain, rebound tenderness Constipation (abdominal mass) Leukocytosis Hematochezia Fever Page | 42 Hepatitis A transmission fecal-oral route poor handwashing, food/water contamination Hepatitis B transmission blood and body fluids Hepatitis C transmission blood and body fluids most common chronic blood borne disease Are alcoholic hepatitis and alcoholic cirrhosis the same? Yes What causes cholecystitis? Obstruction (usually related to GALLSTONES) Symptoms of cholecystitis RUQ pain, fever, fat intolerance, nausea, heartburn, jaundice, steatorrhea, positive Murphy's sign IBS Different from IBD: bloating gas Similar to IBD: abdominal pain cramping changes in BM Dx is CLINICAL recurrent abdominal pain AND pain improves with BM OR onset associated with change in BM frequency or consistency IBD inflammatory bowel disease Different from IBS: Blood in stool Fatigue Weight loss Page | 45 What is Murphy's sign? tenderness on deep palpation of RUQ while patient inspires When a spleen enlarges, how does percussion sounds of the abdomen change? It expands anteriorly, downward and medially (if you palpate too high you will miss an enlarged spleen) Replaces tympany of stomach with dullness of solid organ What is the normal span of the liver and where is that measured? 7cm right mid-clavicular line edge is smooth and palpable 1cm below right costal margin Is hepatitis A acute or chronic? Acute Rarely fatal Where is the spleen located in comparison to the stomach both are in LUQ but spleen is lateral to and behind the stomach, just above the left kidney in the left midaxillary line Why do you auscultate the abdomen first? because percussion and palpation can increase peristalsis, which would give a false interpretation of bowel sounds Hypoactive vs hyperactive bowel sounds Hypoactive <5 per minute Hyperactive >34 per minute abdominopelvic cavity contains both the abdominal and pelvic cavities (2 continuous cavities) abdominal includes RUQ - liver, gallbladder, small intestine, and head of pancreas RLQ - appendix, ascending colon, cecum LUQ - stomach, spleen, body and tail of pancreas LLQ - sigmoid colon, descending colon pelvic cavity is posterior to abdominal cavity and includes ureters bladder pelvic genital organs Page | 46 OLDCARTS for abdominal pain Onset - acute vs. chronic, events surrounding it starting Location & radiation - ask patient to point to it Duration Characteristics - visceral vs. somatic Aggravating & Relieving - attention to body position, association with movement, association with meals, alcohol, medications like NSAIDs, stress, exertion, rest... Treatments and timing - has this occurred before? Severity - scale 1-10 when it is at its worst and now *associated symptoms (N/V, fever...) *PMH - prior abdominal surgeries, recent hospitalization, deliveries, comorbidites like diabetes and digestive disorders, travel outside US, recent antibiotics/axatives/opioids... *PSH - smoking, drug use, STIs Somatic vs. visceral: liver distended from hepatitis visceral Somatic vs. visceral: epigastric pain visceral Somatic vs. visceral: kidney stone somatic Pain from cholecystitis radiates to where? What type of pain is this an example of? Right scapula Referred pain What causes dysphagia? neuro: stroke, Parkinson's, ALS muscular: MG, MS structural: esophageal stricture, Zenkers Acute vs chronic diarrhea acute <14 days (inflammatory, noninflammatory or drug-induced from magnesium, antibiotics, chemo and laxatives) chronic >30 days (IBS, fecal impaction, colon cancer, IBD) What does greasy/oily diarrhea indicate? malabsorption - may related to Celiac, pancreatic insufficiency or small bowel bacterial overgrowth Page | 47 What is melena and what does it indicate? Black tarry stool indicative of an upper GI bleed ulcers, varices, esophagitis What is hematochezia and what does it indicate? Bright red blood in the stool indicative of a lower GI bleed colon cancer, polyps, diverticula, UC, Chron's, infectious diarrhea, hemorrhoids, anal tissue What causes jaundice and what diseases are associated with it? increased bilirubin levels hepatitis, cirrhosis, gallstones, hemolytic anemia What could suprapubic pain indicate? bladder infection over-distention of bladder What urinary symptoms are often associated with BPH? hesitancy and dribbling What urinary symptoms are often associated with UTI? dysuria and urgency Frequency can occur with various volumes of urine, such as... Polyuria (high volume) Oliguira (low volume) Nocturia (waking to urinate) What causes dysuria? -UTI -cystitis -STD -yeast infection -kidney or bladder stones -prostate enlargement -cancers What is urgency? intense and immediate desire to void associated with UTI and urinary colic Page | 50 Percuss L midclavicular to Left Mid-axillary line (Taubes space). Percuss down to dullness (if only tympany, spleen is likely not enlarged) How do you palpate the spleen? With L hand, reach over and around the patient to support and press forward the lower left rib cage and adjacent soft tissue With R hand below the L costal margin, press in toward the spleen Ask pt to take deep breath Try to feel tip of edge of spleen as it comes down to meet your fingertips What causes splenomegaly? portal hypertension, cancer, HIV, amyloidosis, splenic infarct, hematoma, mono When is bladder dull on percussion? When is it palpable? if more than 400-600mL in bladder only palpable if distended Risk factors for AAA over 65, smoking history, 1st degree relative with AAA Symptoms of AAA Palpable abdominal mass with pulsations that is >3cm May be asymptomatic Careful with palpation - consider ultrasound or imaging to assess What are 2 maneuvers for percussing ascites? 1 - patient supine. percuss for dullness outward in several directions from central area of tympany 2 - patient rolls on the side. percuss - dullness should shift more dependent side (percussing to see where the fluid goes) How do you palpate abdominal organs in someone with ascites? brief jabbing movement to displace fluid and allow you to palpate abdominal organ If you suspect a ventral hernia but don't see it, what position should you put the patient in? Have patient raise both legs off the table or perform valsalva maneuver to increase abdominal pressure and make the hernia appear Assessing abdominal wall mass vs intra-abdominal mass abdominal wall mass will stay palpable when patient contracts abdominal muscles by lifting head and shoulders like a sit-up Page | 51 intra-abdominal mass will be obscured with contraction of the muscles Which forms of hepatitis are preventable with vaccines? Who should be vaccinated? A &. B All children and those at high risk Can colorectal cancer be prevented? Yes, that is why screening is important (colonoscopy q10 years age 50-75) Pain from acute cholecystitis vs biliary colic cholecystitis - positive Murphy's. RUQ pain. biliary colic - intermittent RUQ pain that is recurrent and worse with fatty foods What does acute pancreatitis feel like? Steady progressive severe epigastric pain What does bowel obstruction feel like? generalized cramps colicky pain Hallmark of mesenteric ischemia Vague non-specific pain that is out of proportion to examination (associated with underlying thromboembolic disease) What causes urinary frequency? stones infection BPH neural disease tumor Types of incontinence 1. Stress Loss of small amounts of urine from increased abdominal pressure without bladder muscle contraction with laughing, sneezing, or lifting. 2. Urge Inability to stop urine flow long enough to reach the bathroom due to an overactive detrusor muscle with increased bladder pressure, 3. Overflow Page | 52 Urinary retention from bladder over-distention and frequent loss of small amounts of urine due to obstruction of the urinary outlet or an impaired detrusor muscle. Does a palpable liver always indicate hepatomegaly? No, may be displaced downward by COPD (see if vertical span is normal) What are the most urgent conditions that cause acute abdominal pain? peritonitis - risk for hypovolemic shock (fluid in cavity) and septic shock (ruptured abdominal contents in cavity) bleeding - ruptured AAA, ruptured ectopic pregnancy, trauma, PUD ischemia Symptoms of UTI in infant fussy, poor appetite, fever (generalized symptoms) where is mcburneys point? 1/3 the distance from superior iliac spine to umbilicus Key components of prostate and anorectal exam Side lying or standing bent forward Bear down Index finger Only do it if it is needed What are you feeling for on prostate exam? Nodules (r/o malignancy) Enlargement (BPH) Asymmetry (could be normal, BPH or malignancy) Tenderness (prostatitis or abscess possible) Steps of anorectal exam Inspect DRE - sphincter tone - palpate canal - palpate prostate Possible abnormal findings on rectal exam Hemorrhoids (internal or external) Abscess Page | 55 Special maneuver shoulder: Hawkins impingement sign bend elbow at shoulder level and internally rotate Special maneuver shoulder: empty can test arms straight out, thumbs facing down raise arms on resistance tests supraspinatus / rotator cuff Special maneuver shoulder: drop arm test Abduct arm 90 degrees (arm straight out at side of body) Slowly lower arm assesses rotator cuff Special maneuver shoulder: external rotation lag test arm flexed at elbow with palm up, rotate the palm outward and hold the position (positive if the arm lags inward) Special maneuver shoulder: internal rotation lag test back of hand positioned behind lower back examiner pulls hand away from back and asks patient to hold it (positive if arm lags back toward lower back) Special maneuver shoulder: external rotation resistance test palms up bend at elbow resistance on wrists (examiner pushing inward) patient pushes arms outward against examiner What are the peak ages for low back pain? 35 - 50 (under 20 or over 50 could be red flags) What could cause midline spine pain? ligament injury disc herniation degenerative discs vertebral fracture spinal cord mets abscess What could cause off-midline spine pain? muscle strain hip arthritis renal pain sciatica Page | 56 What does low back pain, saddle anesthesia and bowel/bladder incontinence indicate? cauda equina syndrome Characteristics of osteoarthritis >60yo "Wear and tear" locations Heberden nodes (DIP) Bouchard nodes (PIP) Wide base stance Pain worse with activity, stiffness worsens over the day Characteristics of rheumatoid arthritis Tender Painful Stiff (& AM stiffness - may get better over the day) SYMMETRIC Ulnar deviation Pain worse with rest Characteristics of gout uric acid buildup crystal deposits in joints erythema deformities most common in big toe & distal locations What is pseudo gout? calcium crystals instead of urate Muscle strength rating 0 - no muscle contraction / flaccid 1 - barely detectable movement 2 - passive ROM 3 - active motion against gravity but not resistance 4 - active motion against some resistance 5 - active motion against full resistance Lumbar spinal stenosis narrowing of the vertebral foramen in lumbar region spinal nerve roots are compressed can cause pain with standing/walking common with wear and tear / age pain medicine, PT, surgery Page | 57 Epicondylitis "Tennis elbow" The tendons connecting muscles to elbow become inflamed D/t Repetitive wrist and arm motions Pain on the outside of the elbow, and, sometimes, in the forearm and wrist Rest, pain relief medicines and physical therapy. What are concerning findings with neck pain? persistent neck pain after blunt trauma pain that radiates to the arm or causes arm weakness, numbness or paresthesias (could be nerve impingement) consider if stabilization needed Most low back pain is characterized as non-specific could be aching, nerve root entrapment, or spinal stenosis like sciatica Warning signs of underlying disease with low back pain Persist past 6 weeks, fever, weight loss, unremitting pain -> infection (RF: IV drug use, immunosuppression) -> tumor (RF: <18 yo, >50yo, hx cancer) Spinal cord compression - Incontinence, saddle anesthesia, severe or rapidly progressive neuro defect ankylosing spondylitis An inflammatory arthritis affecting the spine and large joints More common among men and usually begins in early adulthood Decreased flexibility, hunched spine, pain in back/spine PT, pain medications Musculoskeletal disorders with systemic manifestations Rheumatoid arthritis Lupus acute septic arthritis Constitutional symptoms - fever, chills, rash, fatigue, anorexia, weight loss, weakness Types of joints Page | 60 Health promotion: fall prevention assess all over 65 for risk for falls risk increases with age, impaired gait, loss of strength, environmental hazards, polypharmacy, comorbid conditions, vision loss, orthostatic hypotension assess gait, strength and balance encourage potentially PT, OT w/ home safety assessment, education, optimizing medication, shoes with traction and vitamin D/calcium Health history musculoskeletal OLDSCARTS point to the pain associated symptoms? (stiffness, fever...) occupation recreational activities mental health, mood, stress other conditions ligaments of knee & tests Anterior cruciate ligament (ACL) - anterior drawer & lachman Posterior cruciate ligament (PCL) - posterior drawer Medial collateral ligament (MCL) - abduction stress test Lateral collateral ligament (LCL) - adduction stress test stress tests push the knee out/in while pushing ankle the opposite way drawer tests knee is bent on table and you sit on the foot and use force to push knee posteriorly and anteriorly to make sure all secure What is the McMurray test? Knee is flexed as much as possible - assess the medial and lateral menisci by internally or externally rotating the knee as you extend it to 90 degrees Nerve pathway (CNS-> PNS) CNS to PNS Impulse conducts via motor/efferent nerves from CNS to muscles and glands. Returns to CNS via sensory/afferent nerve fibers. Motor nerves are either part of the somatic nervous system (skeletal muscles; voluntary control) or the autonomic nervous system (cardiac muscle, smooth muscles, glands; involuntary control). Page | 61 The autonomic nervous system is divided into ________ and ________ sympathetic (fight or flight) parasympathetic (conserves energy, during rest) Assessing mental status Thought process Orientation Mood Appearance Speech Activity Memory Examples of secondary headaches infection (meningitis, sinus) tumor increased ICP (subarachnoid hemorrhage) stroke rebound headache (withdrawal) acute glaucoma concussion trigeminal neuralgia What are the classic signs of subarachnoid hemorrhage? Worst headache of my life thunderclap SNOOP for red flags (headaches) Systemic symptoms (fever, weight loss, fatigue) & secondary risk factors (HIV, immunosuppression) Neuro symptoms (altered LOC, local deficits) Onset (abrupt? thunderclap?) Order (ex. after age 50, concerning for temporal arteritis) Positional, prior headache or papilledema Syncope v lightheadedness v dizziness syncope - loss of consciousness lightheadedness - pre-syncope - legs feel weak, feel like about to pass out Page | 62 dizziness - vertigo, disequilibrium, spinning sensation Migraine headache characteristics POUND (pulsatile, one-day, unilateral, nausea, disabling) +/- aura +/- vision changes, numbness, speech problems, & muscle weakness moderate to severe pain lasts 4-72h more common in females, may start at puberty, most common 35-45 yo *woman over 35 with migraine with aura should avoid combo estrogen OC due to risk for clots Cluster headache characteristics orbital excruciating unilateral attacks multiple times a day for days or months before remission most common in males 20-50yo last 15min - 3h Tension headache characteristics most common tight band or pressure around head gradual onset mild to moderate bilateral pain lasts a few hours most common 35-40yo Components of neuro exam mental status cranial nerves motor sensory reflexes Cranial nerves involved with swallowing, uvula movement, and gag reflex 9 & 10 Cranial nerves involved with speech 5, 7, 9, 10, 12 How to test for balance Page | 65 What is facial palsy and which cranial nerves are involved? flattening of nato-labial folds or drooping lower eyelid CN 7 What is tongue deviation and which cranial nerves are involved? tongue protrudes and deviates to weaker side CN 12 stroke in anterior cerebral artery contralateral leg weakness stroke in middle cerebral artery contralateral arm weakness cortical signs: aphasia-loss of ability to produce/understand language agnosia- loss of ability to recognize thing damage to motor speech (Broca's) and language (Wernickes's) on left side • most common stroke stroke in subcortical MCA motor and sensory deficit without cortical signs like aphasia stroke in posterior cerebral artery visual deficits stroke in basilar artery Locked in syndrome--> B/L paralysis- quadriplegia -> ataxia = loss of muscle control Sexual health history (Ps) Partners - sex & gender Practices Previous history of STI - tested for HIV? Protection from STIs Pregnancy plans Pleasure - pain with intercourse, maintaining erection, orgasm... Who is recommended to have regular screening for chlamydia and gonorrhea? All sexually active women, especially under 24 yo and over 25 yo if at increased risk Page | 66 Insufficient evidence to support screening in men Include behavioral counseling and education on safe sex to decrease risk of STIs Age-related changes of testicles Infants - not descended Adolescent in puberty - enlarged Young adult - no abnormalities Middle age adult - no abnormalities Older adult - decrease in size hypospadias vs epispadias AND what to assess for if present both congenital hypospadias: opening on ventral surface (underside) epispadias: opening on dorsal surface (top side) *assess for psych issues and functional status Assessment of penis, scrotum and testes Inspect - Skin - Prepuce (forskin) - Glans (tip - squeeze to open urethra then inspect for discharge - strip/milk shaft if discharge suspected but not seen) - Scrotum (skin, hair, contour) Palpate - Shaft for tenderness - Each testis for nodules - should be smooth and symmetric - Epididymis (posterior surface, feels like cord, carries sperm) - Hernias - inguinal, femoral - Scrotal masses Male genitalia findings to note - Hypo/epi-spadias - varicoceles (enlarged veins) - hydroceles (fluid accumulation around testicle) - smegma (cheesy white material under foreskin) - chancre / ulcers (r/t STI) - epidermoid cysts (benign white/yellow on scrotum) What is testicular torsion? Page | 67 Testis is abnormally twisted on its spermatic cord which compromises arterial supply and venous drainage leading to ischemia. Surgical emergency! painful, scrotal swelling How to perform testicular self-exam Monthly In shower Gently roll one testicle at a time between fingers - feel spermatic cord on backside, look for lumps/bumps or changes in shape/consistency or pain If a patient complains of a soft, tube-like lump on the back of the left testicle, how would you respond? Assess and provide reassurance - likely the epididymis (feels cordlike) that collects and carries sperm At what age do you assess for menopause? 45-55yo What do you inspect on external female genitalia) mons pubis labia (major/minor) perineum How do you inspect internal female genitalia? speculum (30 degree downward angle) use warm water instead of gel as lube take sample from cervix for Pap smear or STIs Bimanual palpation of the uterus: W/ cervix against the vaginal fingers, the abdominal hand will bring uterus and then ovaries downward to fingers Rectovaginal palpation Page | 70 cancer of the testicle, usually occurring in men 15 to 35 years of age presents as painless nodules with palpable inguinal lymph nodes Syphillis signs painless, smooth genital sores (chancre) Genital warts signs Flesh colored swelling in and around the genital area Can be flat, raised , round, cauliflower shaped and sometimes moist; may be red Can have one single wart or multiple Herpes signs smaller lesions - can be normal, red or crusted Pubic lice sign severe excoriation Symptoms of candidiasis Thick, odorless, cottage cheese-like discharge Symptoms of BV -fishy odor -gray-white discharge, worse after discharge What are myomas? benign uterine tumors (fibroids) that can be mistaken for an ovarian mass if projecting laterally Pelvic inflammatory disease causes acute pelvic pain usually complication of STI can cause ectopic pregnancy consider if had a recent IUD insertion Risk factors for cervical cancer 1. Infection with high-risk HPV strains 2. History of sexually transmitted diseases (chlamydia) 3. Early onset of sexual activity 4. Multiple or high-risk sexual partners 5. Immunosuppression 6. Oral contraceptive use long term 7. Low SES Page | 71 8. Tobacco use 9. More than 3 full term pregnancies GTPAL G gravidity - number of times pregnant - T term - number carried to term 37 weeks or later - P preterm - number delivered 20-36 weeks - A abortions - number ending before 20 weeks L currently living children Gravidity vs. parity & context of multiples Gravidity = total number of pregnancies Parity = total number of deliveries Twins/triplets are considered one gravity and one parity but 2 living children For a woman in late stages of pregnancy, how do you position her on the examining table? Position on left side to prevent vena cava syndrome Naegel's rule for determining due date LMP + 7 days - 3 months + 1 year IF you do not know LMP, you check fundal height (greater than 12-14 weeks pregnant) Fetal movement >18 weeks Heart tones > 12 weeks. Symptoms of pregnancy Absence of menses Breast tenderness Nausea or vomit Fatigue Urinary Freq Pregnancy weight gain recommendations underweight BMI<18.5: 28-40 lbs normal weight BMI<24.9: 25-35 lbs overweight BM<29.9: 15-25 lbs obese BMI>30: 11-20 lbs high risk of listeriosis (food poisoning) Pregnant women 10x more likely to get listeriosis from contaminated foods. Can cause miscarriage, stillbirth, preterm labor, and serious illness or death of newborn. Page | 72 Pregnant patients should avoid: unpasteurized milk raw/undercooked seafood eggs meat pate meat spreads smoked salmon hot dogs cold cuts Vitamin recommendations for pregnancy 600 IU Vit D 1000 mg Ca 50-290 mcg iodine 400 mcg folic acid, 4000 mcg if high risk for neural tube defects 30 mg iron, 60-120 mg for anemia Which vaccines are unsafe during pregnancy? MMR Polio Varicella Live attenuated flu (nasal spray) - inactive flu recommended (all live vaccines) Tdap and Pregnancy Give for EVERY pregnancy regardless of when the last dose was given Can be given anytime, but is best given between 27 - 36 weeks gestation Antibodies are passed to baby to protect them from whooping cough Rubella titer and pregnancy Recall MMR is a live vaccine and cannot give to pregnant women Important to do rubella titer and give vaccine after birth if needed stay away from people with undiagnosed rashes Rh and pregnancy If a mother is Rh(-) and the baby is Rh(+) and the blood mixes, the antibodies may attack the baby's RBC. Hemolytic disease of the fetus and newborn (HDFN) Page | 75 What should you do if you hear a venous hum or puff of air on auscultation of the chest during pregnancy? Nothing - normal due to increased blood flow through breasts Trans pregnancy considerations Trans men (female at birth) - may become pregnant and breastfeed - concern about STIGMA Trans female (male at birth) - may use hormones to be able to breastfeed - CONFIRMS trans identity When are well child visits? 1, 2, 4, 6, 9, 12 months APGAR score a scale of 1-10 to evaluate a newborn infant's physical status (HR, R, muscle tone, reflex irritability, color) at 1 and 5 minutes after birth 0 - absent 2 - normal Gestational age Preterm <37 weeks Term 37-40 weeks Post term >42 weeks Normal birth weight (g) & classifications 2500g <10th percentile = small for gestational age 10-90th percentile = appropriate for gestational age >90th percentile = large for gestational age How do you measure growth in children under and over 2 years? <2y weight, length, head circumference >2y weight, height, BMI Primitive reflexes reflexes, controlled by "primitive" parts of the brain, that disappear during the first year of life Palmar grasp (fingers) Plantar grasp (toes) Rooting reflex Startle reflex Page | 76 Normal HR at birth 120-160 BPM Physical exam - infant fontanelles, sutures facial features eyes follow light, red reflex, blinks (birth) fixates on object (1m) startle to loud noise, calm to soothing noise (brith) nostril patency candida in mouth natal teeth cyanosis of tongue/mouth listen for grunt/wheeze/stridor, WOB auscultate heart - S1 S2 murmurs palpate liver and spleen foreskin doesn't retract more prominent gentialia features in girls d/t estrogen Mental and physical status in kids 1-5y - wellness, mood, nutrition, speech, facial expressions, relationship with parent 6-10y - orientation, factual knowledge, language, number skills, motor skills BMI percentiles for kids <5th percentile underweight >85th percentile risk for overweight >95th percentile overweight -> taller and heavier than 95% of children that age At what age do you start checking blood pressure? 2 years old Is it common to hear heart murmurs in kids? Yes carotid bruit venous hum pulmonary flow Still's murmur Common pediatric skin conditions Page | 77 erythema toxic (pustules, birth) acne seborrhea (red/yellow scales) atopic dermatitis/eczema diaper dermatitis (candida more severe than contact) impetigo (bulla, purulent) verruca - warts tinea - ringworm scabies pityriasis - xmas tress pattern HEEADSSS assessment for teens Home environment Education/employment Eating Activities Drugs Sexuality Suicide Safety Key points for physical exam of adolescent Consider chaperone Eval breast maturation Eval sexual maturity of genitalia Assess for scoliosis Sports exam for adolescents cardiovascular risk factors prior surgeries prior injuries family history inspect standing, active ROM, CN 11, leg raise Pediatric health promotion Wellbeing Oral care Immunizations Screenings Anticipatory guidance (nutrition, safety, exercise, sexuality, parenting, sleep, screen time...) Geriatric syndromes common conditions found in older adults that tend to be multifactorial and do not fall under discrete disease categories, such as falls, delirium, frailty, dizziness, and urinary incontinence